Download as docx, pdf, or txt
Download as docx, pdf, or txt
You are on page 1of 60

1 Tacder NatRes

and Luz Sandoval filed an application for a parcel of land, Lot


7454 originally party of Santiago but had since then been
transferred to Nueva Vizcaya.

A. Governing Law The registration was opposed by Bayaua, Reyes, and the
B. Historical Philippine Cacao and Farm Products.
Background
C. Policy The case went on until on 1981, 20 years after, the Heirs of
Considerations Sandoval, Heirs of Bayaua, and the Bureau of Lands and Bureau
D. Regalian Doctrine of Forest Development entered a compromise agreement, which
effectively distributed parts of lot 7454 among the aforesaid
parties and the counsel of the Heirs of Sandoval as attorney's
Sunbeam Convenience Foods, Inc. vs. CA fees. The compromise agreement was approved by the court and
FACTS: Sunbeam Convenience Foods, Inc. is the recipient of a confirmed the title and ownership of the parties in accordance
Sales Patent issued by the Bureau of Lands over two parcels of with its terms.
land in Bataan. An OCT was thereby issued. The Solicitor-General
filed an action for reversion on the ground that the lots were Having knowledge of the incident, the Solicitor General filed a
forest lands and therefore inalienable. complaint before the court to annul the decision rendered by the
court a quo for being void and made in excess of jurisdiction or
CA ruled, upholding the Solicitor-General's contention. with grave abuse of discretion.

ISSUE: Whether or not land is alienable The Solicitor General contended that the the Heirs of Sandoval
et. al. did not present any evidence to support their claims of
HELD: The SC affirmed. ownership or registration, nor did the government agencies
involve have a y authority to enter into the compromise
Our adherence to the Regalian Doctrine subjects all agricultural, agreement, and finally, that he was not notified of the
timber, and mineral lands to the dominion of the State. Thus, proceedings and so had not opportunity to take part therein.
before any land may be declassified from the forest group and
converted into alienable or disposable land for agricultural As for the Heirs of Sandoval et.al.'s contention, they asseverate
purposes, there must be a positive act from the Government. that the land is not a public land as the possessory information
Even rules on the confirmation of imperfect titles do not apply title in their name and of their predecessors-in-interest, the pre-
unless and until the land classified as forest land is released in an war certification appearing in the Bureau of Archives, and the
official proclamation to that effect so that it may form part of the fact that the proceeding of the registration was brought under
disposable agricultural lands of the public domain. the Torrens act which presupposes an existing title to be
confirmed, are all evidences that the land is a private land.
The mere fact that a title was issued by the Director of Lands
does not confer any validity on such title if the property covered ISSUE: W/N the respondent's evidences can be considered as
by the title or patent is part of the public forest. proof that the lot 7454 is a private land.

RULING: NO. Under the Regalian Doctrine 2 all lands not


otherwise appearing to be clearly within private ownership are
Republic v Sayo presumed to belong to the State. Hence it is that all applicants in
FACTS: The case at bar started at 1961 whe the spouses Casiano land registration proceedings have the burden of overcoming the
presumption that the land thus sought to be registered forms
2 Tacder NatRes

part of the public domain. 3 Unless the applicant succeeds in


showing by clear and convincing evidence that the property Finally, it was error to disregard the Solicitor General in the
involved was acquired by him or his ancestors either by execution of the compromise agreement and its submission to
composition title from the Spanish Government or by possessory the Court for approval. It is, after all, the Solicitor General, who is
information title, or any other means for the proper acquisition of the principal counsel of the Government; this is the reason for
public lands, the property must be held to be part of the public our holding that "Court orders and decisions sent to the fiscal,
domain . 4 The applicant must present competent and acting as agent of the Solicitor General in land registration cases,
persuasive proof to substantiate his claim; he may not rely on are not binding until they are actually received by the Solicitor
general statements, or mere conclusions of law other than General."
factual evidence of possession and title. 5

In the proceeding at bar, it appears that the principal document Seville vs National Development Company
relied upon and presented by the applicants for registration, to FACTS: On January 14, 1980, Calixtra Yap sold to LSBDA a certain
prove the private character of the large tract of land subject of parcel of lot. LSBDA filed a Miscellaneous Sales Application with
their application, was a photocopy of a certification of the the Bureau of Lands covering said lot together with other lots
National Library. But, as this Court has already had occasion to acquired by them. After due notice and investigation, a
rule, that Spanish document cannot be considered a title to Miscellaneous Sales Patent was issued in the name of LSBDA, on
property, it not being one of the grants made during the Spanish the basis of which, an original certificate of title was transcribed
regime, and obviously not constituting primary evidence of in the registration book on their name.
ownership. 6 It is an inefficacious document on which to base any LSBDA subsequently assigned all its rights over the property to
finding of the private character of the land in question. National Development Company. NDC leased the property to
Philippine Associated Smelting and Refining Corporation,
It thus appears that the decision of the Registration Court a PHILPHOS and Lepanto Consolidated Mining.
quo is based solely on the compromise agreement of the parties. On November 29, 1988, the estate of Joaquin Ortega represented
But that compromise agreement included private persons who by judicial administrator Felipe Seville filed a complaint for
had not adduced any competent evidence of their ownership recovery of real property, rentals and damages against the
over the land subject of the registration proceeding. Portions of National Development Company. They argued that they acquired
the land in controversy were assigned to persons or entities who title to the disputed property by acquisitive prescription, because
had presented nothing whatever to prove their ownership of any they and their predecessors in interest had been in possession of
part of the land. The assent of the Directors of Lands and Forest it for more than thirty years.
Development to the compromise agreement did not and could
not supply the absence of evidence of title required of the private ISSUE: WON petitioners’ title, allegedly acquired by them through
respondents. acquisitive prescription, is valid.

As to the informacion posesoria invoked by the private RULING No. Petitioners’ title is not valid.
respondents, it should be pointed out that under the Spanish Under the Regalian Doctrine, all lands of public domain belong to
Mortgage Law, it was considered a mode of acquiring title to the state, which the source of any is asserted right to ownership
public lands, subject to two (2) conditions: first, the inscription of land. All lands not otherwise appearing to be clearly within
thereof in the Registry of Property, and second, actual, public, ownership are presumed to belong to the state.
adverse, and uninterrupted possession of the land for twenty (20) Although it may be true that Section 48, Chapter VIII of the Public
years (later reduced to ten [10] years); but where, as here, proof Land Act provides that those who by themselves or through their
of fulfillment of these conditions is absent, the informacion predecessor-in-interest have been in open, continuous, exclusive,
posesoria cannot be considered as anything more than prima and notorious possession and occupation of agricultural lands of
facie evidence of possession. 7
3 Tacder NatRes

the public domain for at least 30 years could acquire a title 2. W/N the constitutional prohibition against their acquisition
thereto, Section 4 of PD 1073 amended the same to limit its by private corporations or associations applies- NO
application to alienable and disposable lands of the public
domain only. HELD: IAC affirmed Acme Plywood & Veneer Co., Inc
In the case at bar, the Supreme Court reiterated that “unless 1. YES
public land is shown to have been reclassified or alienated to a  already acquired, by operation of law not only a
private person by the state, it remains part of the inalienable right to a grant, but a grant of the Government, for
public domain. Indeed, occupation thereof in the concept of it is not necessary that a certificate of title should
owner, no matter how long, cannot ripen into ownership and be be issued in order that said grant may be
registered as a title. sanctioned by the courts, an application therefore is
Absent any showing that the land has been classified as sufficient
alienable, their possession thereof, no matter how lengthy,  it had already ceased to be of the public domain
cannot ripen into ownership. In other words, the petitioners have and had become private property, at least by
not become the owners of the disputed property. presumption
LSBD, who acquired a Miscellaneous Sales Patent over the  The application for confirmation is mere formality,
subject property, which eventually caused them to have an the lack of which does not affect the legal
Original Certificate of Title for the said land has a valid claim over sufficiency of the title as would be evidenced by the
the property. patent and the Torrens title to be issued upon the
strength of said patent.
 The effect of the proof, wherever made, was not to
Director of Lands vs Intermediate Appellate Court confer title, but simply to establish it, as already
FACTS: conferred by the decree, if not by earlier law
 Acme Plywood & Veneer Co., Inc., a corp. represented by 2. NO
Mr. Rodolfo Nazario, acquired from Mariano and Acer Infiel,  If it is accepted-as it must be-that the land was
members of the Dumagat tribe 5 parcels of land already private land to which the Infiels had a
 possession of the Infiels over the landdates back legally sufficient and transferable title on October
before the Philippines was discovered by Magellan 29, 1962 when Acme acquired it from said owners, it
 land sought to be registered is a private land must also be conceded that Acme had a perfect
pursuant to RA 3872 granting absolute ownership to right to make such acquisition
members of the non-Christian Tribes on land occupied by  The only limitation then extant was that
them or their ancestral lands, whether with the alienable or corporations could not acquire, hold or lease public
disposable public land or within the public domain agricultural lands in excess of 1,024 hectares
 Acme Plywood & Veneer Co. Inc., has introduced
more than P45M worth of improvements
 ownership and possession of the land sought to be
registered was duly recognized by the government when the
Municipal Officials of Maconacon, Isabela United Paracale vs Dela
 donated part of the land as the townsite of
Maconacon Isabela FACTS: United Paracale Mining Company, Inc. and Coco Grove,
 IAC affirmed CFI: in favor of Inc., seek to set aside the Order of dismissal of the case they
filed with the trial court for the ejectment of their respective
ISSUES:
1. W/N the land is already a private land - YES defendants from the mining claims which were allegedly
privately owned by them having been located and perfected
4 Tacder NatRes

under the provisions of the Philippine Bill of 1902 and Act No. claim. To sustain their contention that they can question the
624. award of mining patents to applicants other than them would put
to naught the objectives of P.D. 1214
Petitioners question the constitutionality of P.D. 1214 and unless
it is successfully assailed, the petitioners will be but mere
applicants for the lease of the mining claims involved and would Republic vs Register of Deeds of Quezon
thus have no causes of action against private respondents. Regalian Doctrine
Burden of Proof of private ownership rests on plaintiff
Doctrine of indefeasibility of Torrens title, exception
ISSUE: Does the location and perfection of the mining claims
under the provisions of the Philippine Bill of 1902 and Act No. 624 FACTS: Petitioner was awarded a 17-hectare parcel of land, by
grant ownership to the petitioners. virtue of which he was issued an OCT.

HELD: No. In the case of Santa Rosa Mining Company, Inc. vs. Through an investigation conducted by the Bureau of Lands, it
was found that the free patent acquired by Petitioner was
Leido, Jr. 11, the court said that mere location does not mean
fraudulent. A case for falsification of public documents was filed
absolute ownership over the affected land or the mining claim. It by Petitioner was acquitted of the crime.
merely segregates the located land or area from the public
domain by barring other would-be locators from locating the Subsequently, the Solicitor-General filed a complaint against
same and appropriating for themselves the minerals found Petitioner, praying for the declaration of nullity of the Free Patent
therein. To rule otherwise would imply that location is all that is and the OCT.
needed to acquire and maintain rights over a located mining
Petitioner's main contention was that the land in question was no
claim.
longer within the unclassified public forest land because by the
approval of his application for Free Patent by the Bureau of
Moreover, the court does not agree with petitioners’ contention Lands, the land was already alienable and disposable public
that they cannot be placed in equal footing with those who forfeit agricultural land. He also claimed that the land was a small
all rights by virtue of non-filing of an application within the portion of Lot 5139, an area which had been declared disposable
prescribed period under PD1214. The court said that while those public land by the cadastral court.
who filed their mining lease applications have better rights than
ISSUE: Whether or not the land is alienable and disposable public
those who forfeited all their right by not filing at all, this,
land
however, does not amount to any vested right which could be the
basis for their cause of action against herein private respondents. HELD: Under the Regalian Doctrine, all lands not otherwise
clearly appearing to be privately-owned are presumed to belong
What is precisely waived is their right to the issuance of a mining to the State. Forest lands, like mineral or timber lands which are
patent upon application. This in effect grants the government the public lands, are not subject to private ownership unless they
under the Constitution become private properties. In the absence
power, in the exercise of its sound discretion, to award the patent
of such classification, the land remains unclassified public land
to the applicant most qualified to explore, develop and exploit until released therefrom and rendered open to disposition.
the mineral resources of the country in line with the objectives of
P.D. 463, and not necessarily to the original locator of the mining The task of administering and disposing lands of the public
5 Tacder NatRes

domain belongs to the Director of Lands, and ultimately the parcel of land was classified as a Forest Reserve Area, and hence,
Secretary of Agriculture and Natural Resources. Classification of not capable of private appropriation and occupation. Before any
public lands is, thus, an exclusive prerogative of the Executive land may be declassified from the forest group and converted
Department, through the Office of the President. Courts have no into alienable or disposable land for agricultural or other
authority to do so. purposes, there must be a positive act from the government.
The filing of the public land application is not such positive act.
Thus, in controversies involving the disposition of public The parcel of land retained its public character and it therefore
agricultural lands, the burden of overcoming the presumption of not susceptible to private ownership. As there was no award or
state ownership of lands of the public domain lies upon the grant to Ituralde of the land by free patent or other ways of
private claimant. acquisition, under the concept of jura regalia, the State retains
ownership over the land.
In the present case, Petitioner failed to present clear, positive
and absolute evidence to overcome said presumption and to
support his claim. Republic vs. Court of Appeals and dela Rosa
Facts: Jose dela Rosa filed an application for registration of a
Moreover, the fact the Petitioner acquired a title to the land is of parcel of land on his own behalf and on behalf of his children.
no moment, notwithstanding the indefeasibility of title issued This application was separately opposed by Benguet
under the Torrens System. The indefeasibility of a certificate of Consolidated, Inc. (Benguet) and Atok Big Wedge Corporation
title cannot be invoked by one who procured the same by means (Atok).
of fraud. Fraud here means actual and extrinsic -- an intentional
omission of fact required by law. The petitioners claimed that they have acquired the land from
their parents and that they have been in possession of the land
Petitioner committed fraud by his failure to state that the land ever since. Benguet and Atok opposed on the ground that they
sought to be registered still formed part of the unclassified forest have mineral claims covering the property and had been in
lands. actual, continuous and exclusive possession of the land in
concept of owner.
Ituralde vs Falcasantos
FACTS: In 1986, Ramon Ituralde obtained a 6k ha parcel of land in The trial court denied the application while the Court of Appeals
Basilan Province. Falcasantos, meanwhile, applied with the reversed the decision of the trial court and recognized the claims
Bureau of Lands to grant him the same parcel of land under free of the applicant but subject to the rights of Benguet and Atok
patent. In 1989 the Director of Lands allowed Ituralde to file a respecting their mining claims. In other words, the Court of
public land application for the subject property. 1990 the trial Appeals affirmed the surface rights of the de la Rosas over the
court named Ituralde the owner and possessor of the land. The land while at the same time reserving the sub-surface rights of
CA reversed this decision and set aside the trial court’s decision Benguet and Atok by virtue of their mining claims.
in Ituralde’s favor as the land was found to be forest land, and
therefore, inalienable. Ituralde, in his appeal to the SC, claims Issue: Whether or not the CA's ruling was correct.
that since the Director of Lands allowed him to file a public land
application, the said land was no longer part of public domain. Held: No, the CA was incorrect.

ISSUE: WON the filing of the public land application in 1989 made Art. 437. The owner of a parcel of land is the owner of its surface
the land alienable. NO. and of everything under it, and he can construct thereon any
works or make any plantations and excavations which he may
HELD: The CA is their decision found that since 1951, the subject deem proper, without detriment to servitudes and subject to
6 Tacder NatRes

special laws and ordinances. He cannot complain of the


reasonable requirements of aerial navigation. Heirs of Gozo v. Philippine Union Mission Corp
Facts: Petitioners claim that they are the heirs of Spouses Gozo.
Under the theory of the respondent court, the surface owner will The respondents claim that they own a 5,000 square-meter
be planting on the land while the mining locator will be boring portion of the property. The assertion is based on the 28 February
tunnels underneath. The farmer cannot dig a well because he 1937 Deed of Donation in favor of respondent Philippine Union
may interfere with the operations below and the miner cannot Mission Corporation of the Seventh Day Adventist (PUMCO-SDA).
blast a tunnel lest he destroy the crops above. How deep can the On the date the Deed of Donation is executed in 1937, the
farmer, and how high can the miner, go without encroaching on Spouses Gozo were not the registered owners of the property yet
each other's rights? Where is the dividing line between the although they were the lawful possessors thereof. The land in
surface and the sub-surface rights? question was part of the inalienable public domain

It is a well-known principle that the owner of piece of land has RULING: The Deed of Donation was executed by the Spouses
rights not only to its surface but also to everything underneath Gozo on 28 February 1937, the subject property was part of the
and the airspace above it up to a reasonable height. inalienable public domain. It was only almost after two decades
later or on 5 October 1953 that the State ceded its right over the
The rights over the land are indivisible and that the land itself land in favor of the Spouses Gozo by granting their patent
cannot be half agricultural and half mineral. The classification application and issuing an original certificate of title in their favor.
must be categorical; the land must be either completely mineral Prior to such conferment of title, the Spouses Gozo possessed no
or completely agricultural. right to dispose of the land which, by all intents and purposes,
belongs to the State.
The Regalian doctrine which, as its name implies, is intended for
the benefit of the State, not of private persons. The rule simply Under the Regalian doctrine, which is embodied in Article XII,
reserves to the State all minerals that may be found in public and Section 2 of our Constitution, all lands of the public domain
even private land devoted to "agricultural, industrial, belong to the State, which is the source of any asserted right to
commercial, residential or (for) any purpose other than mining." any ownership of land. All lands not appearing to be clearly
Thus, if a person is the owner of agricultural land in which within private ownership are presumed to belong to the State.
minerals are discovered, his ownership of such land does not give Accordingly, public lands not shown to have been reclassified or
him the right to extract or utilize the said minerals without the released as alienable agricultural land or alienated to a private
permission of the State to which such minerals belong. person by the State remain part of the inalienable public domain.
The flaw in the reasoning of the respondent court is in supposing
that the rights over the land could be used for both mining and The classification of public lands is an exclusive prerogative of
non-mining purposes simultaneously. The correct the executive department of the government and not the Courts.
interpretation is that once minerals are discovered in the In the absence of such classification, the land remains as an
land, whatever the use to which it is being devoted at the unclassified land until it is released therefrom and rendered open
time, such use may be discontinued by the State to to disposition. This is in consonance with the Regalian doctrine
enable it to extract the minerals therein in the exercise of that all lands of the public domain belong to the State and that
its sovereign prerogative. The land is thus converted to the State is the source of any asserted right to ownership in land
mineral land and may not be used by any private party, including and charged with the conservation of such patrimony.
the registered owner thereof, for any other purpose that will
impede the mining operations to be undertaken therein. The All lands not appearing to be clearly within private ownership are
Regalian doctrine then extends not only to land but also to “all presumed to belong to the State. Accordingly, all public lands not
natural wealth that may be found in the bowels of the earth.” shown to have been reclassified or released as alienable
7 Tacder NatRes

agricultural land or alienated to a private person by the State


remain part of the alienable public domain. As already well- FACTS: This is regarding a piece of land which Aniano David
settled in jurisprudence, no public land can be acquired by acquired lawful title thereto, pursuant to his miscellaneous sales
private persons without any grant, express or implied, from the application. After approval of his application, the Director of
government; and it is indispensable that the person claiming title Lands issued an order of award and issuance of sales patent,
to public land should show that his title was acquired from the covering said lot by virtue of which the Undersecretary of
State or any other mode of acquisition recognized by law. To Agriculture and Natural Resources issued a Miscellaneous Sales
prove that the land subject of an application for registration is Patent. The Register of Deeds then issued an original certificate
alienable, the applicant must establish the existence of a positive of title to David.
act of the government such as a presidential proclamation or an
executive order, an administrative action, investigation reports of During all this time, Lee Hong Kok did not oppose nor file any
Bureau of Lands investigators, and a legislative act or a statute. adverse claim.
The applicant may also secure a certification from the
Government that the land applied for is alienable and disposable. ISSUE: Whether or not Lee Hong Kok may question the
government grant
It is beyond question that at the time the gratuitous transfer was
effected by the Spouses Gozo on 28 February 1937, the subject HELD: Only the Government, represented by the Director of
property was part of the public domain and is outside the Lands or the Secretary of Agriculture and Natural Resources, can
commerce of man. It was only on 5 October 1953 that the bring an action to cancel a void certificate of title issued pursuant
ownership of the property was vested by the State to the to a void patent. This was not done by said officers but by private
Spouses Gozo by virtue of its issuance of the OCT pursuant to the parties like the plaintiffs, who cannot claim that the patent and
Homestead Patent granted by the President of the Philippines on title issued for the land involved are void since they are not the
22 August 1953. Hence, the donation of the subject property registered owners thereof nor had they been declared as owners
which took place before 5 October 1953 is null and void from the in the cadastral proceedings after claiming it as their private
very start. property.

As a void contract, the Deed of Donation produces no legal effect The fact that the grant was made by the government is
whatsoever. Quod nullum est, nullum producit effectum. That undisputed. Whether the grant was in conformity with the law or
which is a nullity produces no effect Logically, it could not have not is a question which the government may raise, but until it is
transferred title to the subject property from the Spouses Gozo to raised by the government and set aside, the defendant cannot
PUMCO-SDA and there can be no basis for the church's demand question it. The legality of the grant is a question between the
for the issuance of title under its name. Neither does the church grantee and the government.
have the right to subsequently dispose the property nor invoke
acquisitive prescription to justify its occupation. A void contract is IMPERIUM vs. DOMINIUM:
not susceptible to ratification, and the action for the declaration
of absolute nullity of such contract is imprescriptible. The government authority possessed by the State which is
appropriately embraced int eh concept of sovereignty comes
Imperium vs Dominium under the heading of imperium; its capacity to own or acquire
property under dominium. The use of this term is appropriate
Lee Hong Kok vs. David with reference to lands held by the State in its proprietary
Distinction between IMPERIUM and DOMINIUM character. In such capacity, it may provide for the exploitation
Only the government can question a void certificate of title and use of lands and other natural resources, including their
issued pursuant to a government grant. disposition, except as limited by the Constitution.
8 Tacder NatRes

private land title that existed irrespective of any royal grant from
the State. However, the right of ownership and possession by the
ICCs/IPs of their ancestral domains is a limited form of ownership
IPRA Law and does not include the right to alienate the same.
1. Ancestral Lands and
Ancestral Domains Cariño vs Insular Government
2. Constitutionality of the Facts: Mateo Carino(appellant) filed his petition in the Court of
IPRA LAW Land Registration to be granted a parcel of land consisting of 40
3. Native Title: Exception Hectares,1are and 13 Centares in Baguio, Province of Benguet
to Jura Regalia together with a house erected thereon . It was granted , but the
Government of the Philippines and also on behalf of the United
States averred having taken possession of the property for public
and military purposes, Respondents also asserted that they had
Cruz vs Secretary of DENR title to all the land in the Philippines except to permit private
FACTS: Petitioners Isagani Cruz and Cesar Europa filed a suit for lands to be acquired and no prescription runs against the Spanish
prohibition and mandamus as citizens and taxpayers, assailing crown. The US succeeded the title of Spain through Treaty of
the constitutionality of certain provisions of Republic Act No. Paris and Mateo’s land was not registered and that he had lost
8371, otherwise known as the Indigenous People’s Rights Act of all his rights and now is deemed to be a mere trespasser. Then
1997 (IPRA) and its implementing rules and regulations (IRR). The the Court of First Instance dismissed the application for Mateo
petitioners assail certain provisions of the IPRA and its IRR on the since he did not possessed the land since time immemorial and
ground that these amount to an unlawful deprivation of the the land was property of the Government. The decision was
State’s ownership over lands of the public domain as well as affirmed by the Philippine Supreme Court. Thus the case was
minerals and other natural resources therein, in violation of the brought to the US Supreme Court by virtue of Writ of
regalian doctrine embodied in section 2, Article XII of the Error(general method of bringing cases to this court, an appeal
Constitution. the exception, confined to equity in the main.

ISSUE: Do the provisions of IPRA contravene the Constitution? Mateo Carino in his appeal stated that he is an Igorot of the
Province of Benguet, they have owned the land for more than 50
HELD: No, the provisions of IPRA do not contravene the years before the Treaty of Paris ,they all had been recognized as
Constitution. Examining the IPRA, there is nothing in the law that owners by the Igorots and had inherited or received his land from
grants to the ICCs/IPs ownership over the natural resources his father in accordance with Igorot custom. However, no
within their ancestral domain. Ownership over the natural document of title had been issued from the Spanish Crown
resources in the ancestral domains remains with the State and
the rights granted by the IPRA to the ICCs/IPs over the natural Issue : WON Carino owns the land.
resources in their ancestral domains merely gives them, as
owners and occupants of the land on which the resources are Held : Yes
found, the right to the small scale utilization of these resources, The US Supreme Court reversed the decision of the Philippine
and at the same time, a priority in their large scale development Supreme Court.
and exploitation. Mateo Carino should be granted what he seeks and should not
be deprived of what by the practice and belief of those among
Additionally, ancestral lands and ancestral domains are not part whom he lived, was his property, through a refined interpretation
of the lands of the public domain. They are private lands and of an almost forgotten law of Spain.
belong to the ICCs/IPs by native title, which is a concept of The grant to the plaintiff was the result of the principle of
9 Tacder NatRes

Prescription as mentioned in the royal cedula of 1754 states : “ Whether or not, Proclamation 192 and 423 were valid in
Where such possessors shall not be able to produce title deeds, the absence of proof of ownership of the property by the
it shall be sufficient if they s hall show that ancient possession, government?
as a valid title by prescription” . Moreover, the Decree of June 25,
1880 states that possessors for certain time shall be deemed HELD:
owners, if a cultivated land 20 years, if uncultivated 30 years.
Mateo’s father was the owner of the land by the very terms of Yes. Such proclamations have the character of official
this decree- by Organic Act of July 1, 1902 ,all the property and assertions of ownership, and the presumption is that they have
rights acquired there by the United States are to be administered been issued by right of sovereignty and in the exercise of the
“for the benefit of the inhibitants thereof.” State’s dominical authority. We take not only judicial notice
thereof but accept the same as a valid asseveration of regalian
right over property.
Reservations of lands of the public domain are Civil Law; Ownership; Public Land; Presumption of ownership
valid assertions of Regalian Right issued by right of sovereignty.·Proclamation No. 192 (“RESERVING
Acting Registrars of Land Titles and Deeds of Pasay City FOR THE VETERANS CENTER SITE PURPOSES CERTAIN PARCEL OF
vs. Regional Trial Court Branch 57 Makati LAND OF THE PUBLIC DOMAIN SITUATED IN THE PROVINCE OF
FACTS: RIZAL, ISLAND OF LUZON”) and Proclamation No. 423
On November 5, 1985, Domingo Palomares, as administrator of (“RESERVING FOR MILITARY PURPOSES CERTAIN PARCELS OF THE
the heirs of Delfin Casal, commenced suit with the Regional Trial PUBLIC DOMAIN SITUATED IN THE MUNICIPALITY OF PASIG,
Court, Branch 132, Makati, Metro Manila for declaratory relief, TAGUIG, AND PARAÑAQUE, PROVINCE OF RIZAL, AND PASAY
quieting of title, cancellation of Transfer Certificate of Title No. CITY”) have the character of official assertions of ownership, and
192, and cancellation of entries upon Original Certificate of Title the presumption is that they have been issued by right of
No. 291. sovereignty and in the exercise of the State’s dominical authority.
We take not only judicial notice thereof but accept the same as a
However, during the trial the court found hard evidence on valid asseveration of regalian right over property
record that: (1) the property covered by OCT No. 291 had been
conveyed to the United States of America; (2) it had been later Owners of property over which reconveyance is asserted are
ceded to the Republic of the Philippines; and (3) as a indispensable parties. While there is no presumption that
consequence, OCT No. 291 was cancelled upon final orders of property is Government property until otherwise shown, because
Judge Ostrand. the law recognizes private ownership, thus: Art. 425. Property of
private ownership, besides the patrimonial property of the State,
During the pendency of the case also, Proclamation No. 192 provinces, cities, and municipalities, consists of all property
(“RESERVING FOR THE VETERANS CENTER SITE PURPOSES belonging to private persons, either individually or collectively.
CERTAIN PARCEL OF LAND OF THE PUBLIC DOMAIN SITUATED IN We find hard evidence on record that: (1) the property covered
THE PROVINCE OF RIZAL, ISLAND OF LUZON”) and Proclamation by OCT No. 291 had been conveyed to the United States of
No. 423 (“RESERVING FOR MILITARY PURPOSES CERTAIN PARCELS America; (2) it had been later ceded to the Republic of the
OF THE PUBLIC DOMAIN SITUATED IN THE MUNICIPALITY OF Philippines; and (3) as a consequence, OCT No. 291 was
PASIG, TAGUIG, AND PARAÑAQUE, PROVINCE OF RIZAL, AND cancelled upon final orders of Judge Ostrand.
PASAY CITY”) were issued by the government.

ISSUE: Classification of Lands


a) Constitution (Sec 3-5 Art XII) Constitution (Sec 3-5 Art XII)
b) Public Land Act
c) Classification of Public Lands Open to Disposition
d) Classification of Lands an Executive Privilege
e) Public Land vs Government
10 Tacder NatRes

Palomo v. Court of Appeals Republic vs Candy Maker Inc


FACTS: In 1913, some 440,530 square meters of land in Albay FACTS: On April, 29, 1999, Antonia, Eladia, and Felisa, all
were reserved for provincial park purposes by virtue of EO No. surnamed Cruz, executed a Deed of Absolute Sale in favor of
40. Of said area, 15 parcels of land were registered in the name Candy Maker, Inc. for a parcel of land located below the
of Diego Palomo by the Court of First Instance. reglementary lake elevation of 12.50m, about 900 meters away
the Laguna de Bay. Candy Maker, Inc. as applicant, filed an
In 1937, Diego Palomo donated these lands to his heirs Ignacio application with the MTC of Taytay, Rizal for registration of its
and Carmen Palomo. Claiming that the aforesaid original alleged title over the lot. The CENRO of Antipolo City declared the
certificates of title were lost during the Japanese occupation, land to fall within the alienable and disposable zone. On the other
Ignacio Palomo filed a petition for reconstitution with the Court of hand, the Land Registration Authority recommended the
First Instance of Albay on May 30, 1950. The Register of Deeds of exclusion of lot no. 3138-B on the ground that it is a legal
Albay issued Transfer Certificates of Title Nos. 3911, 3912, 3913 easement and intended for public use, hence, inalienable and
and 3914 sometime in October 1953. indisposable. On July 2001, the Republic of the Philippines, the
Laguna Lake Development Authority (LLDA) filed its opposition
On July 10, 1954, President Magsaysay issued Proclamation No. which alleged that the lot subject of the application for
47 converting the area covered by EO 40 into the Tiwi Hot Spring registration may not be alienated and disposed since it is
National Park. The Palomos contended that they have been in considered part of the Laguna Lake Bed, a public land within, its
possession of the subject lands and have introduced jurisdiction.
improvements thereon.
ISSUE: Whether the property subject of the amended application
is alienable and disposable property of the State, and if so,
ISSUE: Were the Original Certificate of Titles issued to the
whether respondent adduced the requisite quantum of evidence
petitioners valid? –NO
to prove its ownership over the property?
HELD Before the Treaty of Paris in 1899, the lands, whether
Held: The property subject of this application was alienable and
agricultural, mineral, or forest were under the exclusive
disposable public agricultural land. However, respondent failed to
patrimony and dominion of the Spanish crown.Private ownership
prove that it possesses registrable title over the property. The
of land could only be acquired through royal concessions which
statute of limitations with regard to public agricultural lands does
were documented in various forms, such as (1) Titulo Real or
not operate against the statute unless the occupant proves
Royal Grant," (2) Concession Especial or Special Grant, (3) Titulo
possession and occupation of the same after a claim of
de Compra or Title by Purchase and (4) Informacion Posesoria or
ownership for the required number of years to constitute a grant
Possessory Information title obtained under the Spanish
from the State. A mere casual cultivation of portions of the land
Mortgage Law or under the Royal Decree of January 26, 1889.
by the claimant does not constitute sufficient basis for a claim of
ownership, such possession is not exclusive and notorious as to
The decision of the CFI relied upon by petitioners were not signed
give rise to presumptive grant from the state. In light of the
by the judge but were merely certified copies of notification to
foregoing, the petition of the Republic of the Philippines is
Diego Palomo bearing the signature of the Clerk of Court.
granted.

More importantly, the lands in question were not classified as


alienable lands. Since the lands were made part of a reservation
for provincial park purposes, they form part of the forest zone.
Chavez vs Public Estates Authority
Thus, they cannot be the valid subject of alienation.
FACTS: President Marcos through a presidential decree
created PEA, which was tasked with the development,
11 Tacder NatRes

improvement, and acquisition, lease, and sale of all kinds of have been reclaimed, and the rest of the area are still submerged
lands. The then president also transferred to PEA the foreshore areas forming part of Manila Bay. Further, it is provided that
and offshore lands of Manila Bay under the Manila-Cavite Coastal AMARI will reimburse the actual costs in reclaiming the areas
Road and Reclamation Project. of land and it will shoulder the other reclamation costs to be
incurred.
Thereafter, PEA was granted patent to the reclaimed areas
of land and then, years later, PEA entered into a JVA with AMARI The foreshore and submerged areas of Manila Bay are part of the
for the development of the Freedom Islands. These two lands of the public domain, waters and other natural
entered into a joint venture in the absence of any public resources and consequently owned by the State. As such,
bidding. foreshore and submerged areas shall not be alienable unless
they are classified as agricultural lands of the public domain.
Later, a privilege speech was given by Senator The mere reclamation of these areas by the PEA doesn’t convert
President Maceda denouncing the JVA as the grandmother of all these inalienable natural resources of the State into alienable
scams. An investigation was conducted and it was concluded and disposable lands of the public domain. There must be a law
that the lands that PEA was conveying to AMARI were lands of or presidential proclamation officially classifying these
the public domain; the certificates of title over the reclaimed lands as alienable and disposable if the law has
Freedom Islands were void; and the JVA itself was illegal. This reserved them for some public or quasi-public use.
prompted Ramos to form an investigatory committee on the
legality of the JVA.

Petitioner now comes and contends that the government


stands to lose billions by the conveyance or sale of the
reclaimed areas to AMARI. He also asked for the full
disclosure of the renegotiations happening between the parties. Public Land Act
Classification of Public Lands Open to Disposition

ISSUE: W/N stipulations in the amended JVA for the transfer Alba vs Court of Appeals
to AMARI of the lands, reclaimed or to be reclaimed, violate the FACTS:
Constitution. In 1958, Lachica filed an application for title to a 4,485 sq.m.
parcel of land which he had acquired through purchase dating
back to 1945. However, some parties filed their opposition
thereto invoking that they are owners of some parts of land
HELD: The ownership of lands reclaimed from foreshore and
submerged areas is rooted in the Regalian doctrine, which holds within the total area applied for. The lower court ruled in favor
of Lachica and held, among others, that Lachica had been in
that the State owns all lands and waters of the public domain.
actual, open and continuous possession of the subject land in the
concept of owner since 1945 and thus had acquired it through
The 1987 Constitution recognizes the Regalian doctrine. It prescription.
declares that all natural resources are owned by the State
and except for alienable agricultural lands of the public
ISSUE:
domain, natural resources cannot be alienated.
Did Lachica acquire the subject property by prescription?
The Amended JVA covers a reclamation area of 750 hectares.
RULING:
Only 157.84 hectares of the 750 hectare reclamation project
The Court ruled on the negative.
12 Tacder NatRes

to constitute a grant from the State.


Based from the facts and evidences presented, it was proven that
Lachicha only had a title to a 620sq.m. portion of the total area. Under (b), Sec. 48, CA 141, confirmation of an imperfect title to a
Prescription cannot be had on the remaining area as the Court public domain requires that:
held that the law applicable in this case is Sec. 48 of CA 141 (wc 1. There be an open, continuous, exclusive and notorious
deals with registration of lands of public domain) and not Sec. 19 possession and occupation of agricultural lands of the
of Act 496 (wc deals with registration of private lands) and with public domain;
which the lower courts had relied on. 2. It should be under a bona fide claim of ownership; and
3. possession should be for at least thirty years immediately
The law in force at the time an action accrues is what governs preceding the filing of the application for confirmation of
the proceeding consistent with the fundamental dictum that laws title except when prevented by war or force majeure
shall have no retroactive effect, unless the contrary is proved. In
this case, the lower courts relied on the provisions on prescription In this case, Lachica had not yet satisfied the requirement of the
with the assumption that the subject property is a private land. 30 years possession, hence, prescription cannot be granted in
However, the application for registration should be that of a favor of him.
judicial confirmation of an imperfect title considering that the
land is presumed under the Regalian Doctrine to be part of the
public domain.

Public lands are classified into


1. alienable or disposable lands which includes agricultural
lands and Classification of Land an Executive Privilege
2. inalienable or non-disposable lands or those not
susceptible of private appropriation which includes Timber Republic vs Imperial
lands and Mineral lands. For purposes of administration FACTS: On September 12, 1917, the late Elias Imperial was
and disposition, the lands of the public domain classified issued Original Certificate of Title (OCT) 408 (500) pursuant to
as "disposable" or "alienable" are further sub-classified Decree No. 55173 of then Court of First Instance of Albay. OCT
into No. 55173 was subdivided and further subdivided resulting in the
issuance of several titles, which are now the subjects of herein
a) Agricultural; petition in the name of private respondents. Petitioner Republic
b) Residential, commercial, industrial or for similar of the Philippines filed a case with the trial court to judicially
productive purposes; declare the Transfer Certificates of Title (TCT) issued to herein
c) Educational, charitable or other similar purposes, private respondents null and void on the ground that the subject
and land, on which the OCT was based, has the features of a
d) Reservations for town sites and for public and foreshore land based on an investigation conducted by the DENR,
quasi-public purposes. Region V, Legazpi City. Respondents, on the other hand contend
that Director of Lands found Jose Baritua's land covered by TCT
Possession of public agricultural land, however long the period No.18655, which stemmed from OCT 408(500), to be "definitely
may have extended, never confers title thereto upon the outside of the foreshore area."
possessor and it is because the statute of limitations with regard
to public agricultural land does not operate against the State, Within the time for pleading, private respondents EANCRA
unless the occupant can prove possession and occupation of the Corporation, Lolita Alcazar and Salvador Alcazar filed their
same under claim of ownership for the required number of years answer with cross-claim, while the rest, namely, Felix S. Imperial,
13 Tacder NatRes

Feliza S. Imperial, Elias S. Imperial and Miriam S. Imperial filed a investigation of the DENR, Region V, Legazpi City, disclosed that
motion to dismiss. They contended that the adjudication by the the land covered by OCT No. 408 (500) from whence the titles
cadastral court is binding against the whole world including the were derived "has the features of a foreshore land." The
Republic since the cadastral proceedings are in rem and the contradictory views of the Director of Lands and the DENR,
government itself through the Director of Lands instituted the Region V, Legazpi City, on the true nature of the land, which
proceedings and was a direct and active participant therein. contradiction was neither discussed nor resolved by the RTC,
Petitioner, through the Office of the Solicitor General, filed an cannot be the premise of any conclusive classification of the land
objection to the motion to dismiss. After hearing the motion to involved.
dismiss, the trial court dismissed the complaint on the ground
that the judgment rendered by the cadastral court in G.R. Cad. The need, therefore, to determine once and for all whether the
Rec. No. 88 and the Courts resolution in the petition to quiet title, lands subject of petitioner's reversion efforts are foreshore lands
G.R. 85770, both decreed that the parcel of land covered by OCT constitutes good and sufficient cause for relaxing procedural
No. 408 (500) was not foreshore. Petitioner appealed to the Court rules and granting the third and fourth motions for extension to
of Appeals. The appellate court denied petitioners motion for file appellant's brief. Petitioner's appeal presents an exceptional
reconsideration for lack of merit and for failure to file the circumstance impressed with public interest and must then be
appellants brief within the extended period granted to petitioner. given due course.

Hence, the present petition. In the case at bar, the need to determine once and for all
whether the lands subject of petitioners reversion efforts are
ISSUE: Whether or not the petition should be granted. foreshore lands constitutes good and sufficient cause for relaxing
the procedural rules and granting the third and fourth motions for
HELD: Yes. extensions to file appellants brief. Petitioner Republics appeal
At the core of the controversy is whether the parcels of land in presented an exceptional circumstance impressed with public
question are foreshore lands. Foreshore land is a part of the interest which in the Courts discretion must be given due course.
alienable land of the public domain and may be disposed of only
by lease and not otherwise. It was defined as "that part (of the
land) which is between high and low water and left dry by the Director of Lands vs CA and Bisnar
flux and reflux of the tides." It is also known as "a strip of land Facts: In their joint application for registration of title to two (2)
that lies between the high and low water marks and, is parcels of land filed on July 20, 1976, the applicants Ibarra and
alternatively wet and dry according to the flow of the tide." Amelia Bisnar claimed to be the owners in fee simple of Lots 866
and 870 of the Pilar Cadastra, containing an area of 28 hectares
The classification of public lands is a function of the executive (284,424 sq.m.) and 34 hectares (345,385 sq.m.) situated in
branch of government, specifically the director of lands (now the barrio Gen. Hizon, Municipality of President Roxas, Province of
director of the Lands Management Bureau). The decision of the Capiz (p. 14, Rollo). The applicants alleged that they inherited
director of lands when approved by the Secretary of the those parcels of land (p. 41, Rollo) and they had been paying the
Department of Environment and Natural Resources (DENR) as to taxes thereon (p. 40, Rollo). The Director of Lands and the
questions of fact is conclusive upon the court. Director of the Bureau of Forest Development, opposed the
application. On February 24, 1977, the applicants filed an
There is allegedly a conflict between the findings of the Director amended application, which was approved on March 14, 1977,
of Lands and the DENR, Region V, in the present case. and included the following allegation: "Should the Land
Respondents contend that the Director of Lands found Jose Registration Act invoked be not applicable to the case, they
Baritua's land covered by TCT No.18655, which stemmed from hereby apply for the benefits of Chapter 8, Commonwealth Act
OCT 408(500), to be "definitely outside of the foreshore area." 141, as amended, as they and their predecessors-in-interest
Petitioner, on the other hand, claims that subsequent
14 Tacder NatRes

have been in possession of the land as owners for more than fifty imperfect title cases, the applicant shoulders the burden of
(50) years." (p. 16, Rollo.) After hearing, the trial court ordered proving that he meets the requirements of Section 48,
the registration of the title of the lots in the names of the Commonwealth Act No. 141, as amended by Republic Act 1942.
applicants, herein private respondents. On appeal, the Appellate He must overcome the presumption that the land he is applying
Court affirmed the trial court's decision. for is part of the public domain but that he has an interest therein
sufficient to warrant registration in his name because of an
Issue: Whether the lots in question may be registered under imperfect title such as those derived from old spanish grants or
Section 48(b) of CA 141 that he has had continuous, open and notorious possession and
occupation of agricultural lands of the public domain under a
Ruling: In the case of Bureau of Forestry vs. Court of Appeals, 153 bona fide claim of acquisition of ownership for at least thirty (30)
SCRA 351, we ruled: years preceding the filing of his application." (Heirs of
Amunategui vs. Director of Forestry, 126 SCRA 69.)
"As provided for under Section 6 of Commonwealth Act 141,
which was lifted from Act 2874, the classification or ALIENABLE OR DISPOSABLE PUBLIC LANDS; CLASSIFICATION OR
reclassification of public lands into alienable or disposable, RECLASSIFICATION THEREOF AS PREROGATIVE OF THE
mineral or forest lands is now a prerogative of the Executive EXECUTIVE DEPARTMENT. "As provided for under Section 6 of
Department of the government and not the courts. With these Commonwealth Act 141, which was lifted from Act 2874, the
rules, there should be no more room for doubt that it is not the classification or reclassification of public lands into alienable or
court which determines the classification of lands of the public disposable, mineral or forest lands is now a prerogative of the
domain into agricultural, forest or mineral but the Executive Executive Department of the government and not the courts.
Branch of the government, through the Office of the President. With these rules, there should be no more room for doubt that it
Hence, it was grave error and/or abuse of discretion for is not the court which determines the classification of lands of
respondent court to ignore the uncontroverted facts that (1) the the public domain into agricultural, forest or mineral but the
disputed area is within a timberland block, and (2) as certified to Executive Branch of the government, through the Office of the
by the then Director of Forestry, the area is needed for forest President.
purposes." (pp. 21-22, Rollo.)
POSITIVE ACT OF THE GOVERNMENT NEEDED TO DECLASSIFY
It bears emphasizing that a positive act of the government is LAND INTO ALIENABLE LAND FOR AGRICULTURAL OR OTHER
needed to declassify land which is classified as forest and to PURPOSES. — It bears emphasizing that a positive act of the
convert it into alienable or disposable land for agricultural or government is needed to declassify land which is classified as
other purposes (Republic vs. Animas, 56 SCRA 499). Unless and forest and to convert it into alienable or disposable land for
until the land classified as forest is released in an official agricultural or other purposes (Republic v. Animas, 56 SCRA 499).
proclamation to that effect so that it may form part of the Unless and until the land classified as forest is released in an
disposable agricultural lands of the public domain, the rules on official proclamation to that effect so that it may form part of the
confirmation of imperfect title do not apply. disposable agricultural lands of the public domain, the rules on
confirmation of imperfect title do not apply.
Thus, possession of forest lands, however long, cannot ripen into
private ownership. 1. the classification or reclassification of public lands into
alienable or disposable agricultural land, mineral land or forest
Section 48 (b) of Commonwealth Act No. 141, as amended, land is a prerogative of the Executive Department of the
applies exclusively to public agricultural land. Forest lands or government and not of the courts;
areas covered with forests are excluded (p. 26, Rollo). 2. that possession of forest lands, no matter how long, cannot
ripen into private ownership; and
We reiterate our ruling in Amunategui that: "In confirmation of 3. that an applicant for registration of title has the burden of
15 Tacder NatRes

proving that he meets the requirements of Section 48 of Com. much real property belonging to the Government which is not
Act No. 141, as amended. affected by statutes for the settlement, prescription or sale of
public lands. Examples in point are properties occupied by public
buildings or devoted to municipal or other governmental uses.

Public Land vs Government Land It is settled that the general legislation of Congress in respect to
public lands does not extend to tide lands. It provided that the
Montano vs Insular Government scrip might be located on the unoccupied and unappropriated
Facts: Isabelo Montano presented a petition to the Court of Land public lands. As said in Newhall vs. Sanger (92 U.S. 761, 763.) A
Registration for the inscription of a piece of land in the barrio of marshland which is inundated by the rise of tides belong to the
Libis, municipality of Caloocan, used as a fishery having a State and is not susceptible to appropriation by occupation, has
superficial area of 10,805 square meters, and bounded as set out no application in the present case inasmuch as in said case the
in the petition; its value according to the last assessment being land subject matter of the litigation was not yet titled
$505.05, United States currency. This petition was opposed by
the Solicitor-General in behalf of the Director of Lands, and by
the entity known asObras Pias de la Sagrada Mitra, the former on
the ground that the land in question belonged to the Government Modes of Disposition
of the United States, and the latter, that it was the absolute a) Homestead Settlement
owner of all the dry land along the eastern boundary of the said b) Sale of Public Agricultural Land
fishery. The Court of Land Registration in its decision of c) Lease
December 1, 1906, dismissed the said oppositions without costs d) Confirmation of Imperfect Title
and decreed, after a general entry by default, the adjudication e) Non-registrable Land
and registration of the property described in the petition, in favor f) Administrative Legalization
of Isabelo Montano y Marcial. From this decision only counsel for g) Special Patents
the Director of Public Lands appealed to this court. and precisely
Isabelo Montano sought title thereon on the strength of 10 years'
occupation pursuant to paragraph 6, section 5 of Act 926 of the Modes of Disposition
Philippine Commission

Issue: Whether or not the land in question can be acquired by


Montano Lee Hong Hok vs David
DOCTRINE: Imperium distinguished from dominum. The
Held: Accordingly, "government land" and "public domain" are government authority possessed by the state which is
not synonymous items. The first includes not only the second, appropriately embraced in the concept of sovereignty comes
but also other lands of the Government already reserved or under the heading of imperium; and its capacity to own or
devoted to public use or subject to private right. In other words, acquire property under dominium. The use of this term is
the Government owns real estate which is part of the "public appropriate with reference to lands held by the state in its
lands" and other real estate which is not part thereof. proprietary character. In such capacity, it may provide for the
Government property was of two kinds — first, that of public use exploitation and use of lands and other natural resources,
or service, said to be of public ownership, and second, that of including their disposition, except as limited by the Constitution.
having a private character or use. (Civil Code, arts. 339 and 340.)
Lands of the first class, while they retain their public character FACTS: Petitioners Lee Hong Hok et al. claim that the Torrens Title
are inalienable. Those of the second are not. Therefore, there is of Respondent David over the disputed land (which is part of the
16 Tacder NatRes

Naga Cadastre) should be declared null and void. The CA found


no legal justification for nullifying the right of David over the In discussing the concept of jura regalia, the Supreme Court said:
disputed land arising from the grant made in his favor by
appropriate public officials. “As there are overtones indicative of skepticism, if not of outright
rejection, of the well-known distinction in public law between the
David had acquired lawful title over said land. The Director of government authority possessed by the state which is
Lands awarded him an order for issuance of a sales patent appropriately embraced in the concept of sovereignty, and its
pursuant to his miscellaneous sales application. Subsequently, on capacity to own or acquire property, it is not inappropriate to
the basis of such order, the Undersecretary of Agricultural and pursue the matter further. The former comes under the heading
Natural Resources issued a Miscellaneous Sales Patent and an of imperium and the latter of dominium. The use of this term is
OCT was issued by the Register of Deeds of Naga City in favor of appropriate with reference to lands held by the state in its
the respondent. proprietary character. In such capacity, it may provide for the
exploitation and use of lands and other natural resources,
ISSUE: WON the State can dispose of lands which have not including their disposition, except as limited by the Constitution.
passed into private ownership. Dean Pound did speak of the confusion that existed during the
medieval era between such two concepts, but did note the
HELD: In this case the land in question is not private property as existence of res publicae as a corollary to dominium. As far as
the Director of Lands and the Secretary of Agriculture and the Philippines was concerned, there was a recognition by Justice
Natural Resources have always sustained the public character Holmes in Cariño v. Insular Government, a case of Philippine
thereof for having been formed by reclamation. origin, that "Spain in its earlier decrees embodied the universal
feudal theory that all lands were held from the Crown . . ."
It is well-settled "that no public land can be acquired by private
persons without any grant, express or implied, from the That was a manifestation of the concept of jura regalia, which
government." It is indispensable then that there be a showing of was adopted by the present Constitution, ownership however
a title from the state or any other mode of acquisition recognized being vested in the state as such rather than the head thereof.
by law. What was stated by Holmes served to confirm a much more
extensive discussion of the matter in the leading case of
In the case at bar, a Miscellaneous Sales Patent and OCT was Valenton v. Murciano, decided in 1904. One of the royal decrees
issued in favor of respondent David by competent public officials. cited was incorporated in the Recopilacion de Leyes de las Indias
He had acquired the grant and title legally. The notices regarding in these words: "We having acquired in sovereignty over the
the auction sale of the land were published, the actual sale and Indies, and all lands, territories, and possessions not heretofore
award thereof to David were not clandestine but open and public ceded away by our royal predecessors, or by us, or in our name,
official acts of an officer of the Government. The application was still pertaining to the royal crown and patrimony, it is our will that
merely a renewal of his deceased wife's application, and the said all lands which are held without proper and true deeds of grant
deceased occupied the land since 1938. be restored to us according as they belong to us, in order that
after reserving before all what to us or to our viceroys,
(The first paragraph of Section 2, Article XII says that “all lands of audiencias, and governors may seem necessary for public
the public domain x x x and other natural resources are owned squares, ways, pastures, and commons in those places which are
by the state,”) peopled, taking into consideration not only their present
condition, but also their future and their probable increase, and
A grant by the government through duly competent public after distributing to the natives what may be necessary for tillage
officials cannot be disregarded on the premise that land not and pasturage, confirming them in what they now have and
passing into private ownership may not be disposed of by the giving them more if necessary, all the rest of said lands may
state.
17 Tacder NatRes

remain free and unencumbered for us to dispose of as we may alienable portion of the public domain.
wish." The Trial Court ordered registration of the subject land in favor of
the Valerianos. This was affirmed by the CA which said in part
It could therefore be affirmed in Montano v. Insular Government that “since the subject property is entirely devoted to fishpond
that "as to the unappropriated public lands constituting the purposes, it cannot be categorized as part of forest lands.”
public domain the sole power of legislation is vested in Congress,
. . ." They continue to possess that character until severed Issue: WON the courts can reclassify the subject public land.
therefrom by state grant. Where, as in this case, it was found by
the Court of Appeals that the disputed lot was the result of Held: Courts cannot reclassify... it’s beyond their competence and
reclamation, its being correctly categorized as public land is jurisdiction.
undeniable. What was held in Heirs of Datu Pendatun v. Director The classification of public lands is an exclusive prerogative of
of Lands finds application. Thus: "There being no evidence the Executive Department of the Government (Bureau of Forest
whatever that the property in question was ever acquired by the Development) and not of the Courts. In the absence of such
applicants or their ancestors either by composition title from the classification, the land remains as unclassified land until it is
Spanish Government or by possessory information title or by any released therefrom and rendered open to disposition.
other means for the acquisition of public lands, the property must
be held to be public domain." For it is well-settled "that no public Since the subj property is still unclassified, whatever possession
land can be acquired by private persons without any grant, Applicants (Valeriano) may have had, and, however long, cannot
express or implied, from the government." It is indispensable ripen into private ownership. The conversion of the subj property
then that there be a showing of a title from the state or any other into a fishpond by Applicants does not automatically render the
mode of acquisition recognized by law. The most recent property as alienable and disposable.
restatement of the doctrine, found in an opinion of Justice J.B.L.
Reyes, follows: "The applicant, having failed to establish his right The recommendation of the District Forester for release of subj
or title over the northern portion of Lot No. 463 involved in the property from unclassified region is not the ultimate word on the
present controversy, and there being no showing that the same matter.
has been acquired by any private person from the Government,
either by purchase or by grant, the property is and remains part 1. CONSTITUTIONAL LAW; LANDS OF THE PUBLIC DOMAIN;
of the public domain." To repeat, the second assignment of error CLASSIFICATIONS THEREOF, A PREROGATIVE OF THE EXECUTIVE.
is devoid of merit.” — The classification of public lands is an exclusive prerogative of
the Executive Department of the Government and not of the
Director of Land vs CA and Valeriano Courts.
Facts: The land in question is situated in Obando, Bulacan. It
adjoins the Kailogan River and private respondent Valeriano have 2. ID.; ID.; ID.; UNCLASSIFIED LAND, NOT SUBJECT TO
converted it into a fishpond. DISPOSITION. — In the absence of such classification, the land
In their application in 1976, private respondents claimed that remains as unclassified land until it is released therefrom and
they are the co-owners in fee simple of the land partly through rendered open to disposition (Sec. 8, Commonwealth Act No.
inheritance and partly by purchase and that; it is not within any 141, as amended; Vide Yngson v. Secretary of Agriculture and
forest or military reservation. Natural Resources, 123 SCRA 441 [1983]; Republic v. Court of
The Republic of the Phil., represented by the Dir of the Bureau of Appeals, 99 SCRA 742 [1980]. This should be so under time-
Forest Development, opposed the application on the principal honored Constitutional precepts. This is also in consonance with
ground that the land applied for is WITHIN THE UNCLASSIFIED the Regalian doctrine that all lands of the public domain belong
REGION of Obando, Bulacan and that such area are denominated to the State (Secs. 8 & 10, Article XIV, 1973 Constitution, and
as FOREST LANDS-do not form part of the disposable and that the State is the source of any asserted right to ownership in
18 Tacder NatRes

land and charged with the conservation of such patrimony 30, 1997 decision against the Menguito’s –the petitioners, with
(Republic v. Court of Appeals, 89 SCRA 648 [1979]). promulgated resolution 10-Mos. later reversing the decision of
the Regional Trial Court of Pasig City. The RTC decision confirmed
3. ID.; ID.; ID.; ID.; CASE AT BAR. — The recommendation of the the application for the titling of the parcel of land with aggregate
District Forester for release of subject property from the area of 2112 sqm located at Brgy Ususan, Taguig, Metro Manila,
unclassified region is not the ultimate word on the matter. And in favor of the Menguito’s.
the fact that BF Map LC No. 637 dated March 1, 1927 showing
subject property to be within the unclassified region was not The application of TCT was brought by the operation of the Land
presented in evidence will not operate against the State Registration of Authority as amended by the Property
considering the stipulation between the parties and under the Registration Decree No. 1529 proceeding to declare:
well-settled rule that the State can not be estopped by the
omission, mistake or error of its officials or agents (Republic v. a) That its applicants –the Menguito’s, are owners, in fee
Court of Appeals, 89 SCRA 648 [1979]), if omission there was, in simple, these 11 parcels of land.
fact. Since the subject property is still unclassified, whatever b) Listing the applicants lot title numbers, attendant
possession Applicants may have had, and, however long, cannot documents given with their respective Technical
ripen into private ownership. (Adorable v. Director of Lands, 107 Descriptions.
Phil. 401; Director of Forestry v. Muñoz, 23 SCRA 1184-1216
[1968]; Director of Lands v. Abanzado, 65 SCRA 5 [1975]; For the said application, the RTC of Pasig issued a notice of its
Republic v. Court of Appeals, 89 SCRA 648, 656 [1979]). initial hearing against the whole world publishing the same at
Abante Tabloid on April 5, 1989.
4. ID.; ID.; ID.; ID.; CADASTRAL SURVEY OF MUNICIPALITY DOES
NOT AUTOMATICALLY RELEASE ALL LANDS THEREIN AS
Seven days earlier, however, March 30, 1989, the Office of
ALIENABLE. — While it may be that the Municipality of Obando
Solicitor General, filed its contention as:
has been cadastrally surveyed in 1961, it does not follow that all
1. Applicants nor its predecessor’s in interest were neither in
lands comprised therein are automatically released as alienable.
open, continuous, exclusive, adverse and notorious
A survey made in a cadastral proceeding merely identifies each
possession or occupation of the land they applied for
lot preparatory to a judicial proceeding for adjudication of title to
since 1945.
any of the lands upon claim of interested parties. Besides, if land
2. Applicants don’t have competent and sufficient evidence
is within the jurisdiction of the Bureau of Forest Development, it
of bonafide acquisition –without open, uninterrupted-
would be beyond the jurisdiction of the Cadastral Court to
continuous, exclusive, adverse or notorious occupation of
register it under the Torrens System.
the lot in the concept of the owner and so appearing not
genuine or indicative of pretended possession.
5. ID.; ID.; ID.; ID.; CONVERSION OF UNCLASSIFIED LAND INTO A
3. Titling from fee simple with Spanish grant title were not
FISHPOND DOES NOT MAKE LAND ALIENABLE. — The conversion
anymore available after Feb 16, 1976 as required by PD
of subject property into a fishpond by Applicants, or the alleged
892.
titling of properties around it does not automatically render the
4. That the said parcel of land applied for is part of the public
property as alienable and disposable. Applicants’ remedy lies in
domain and belonging to the Republic of the Philippines –
the release of the property from its present classification.
and not subject to private appropriation.

The OSG thus stated its valid opposition on the presented


Menguito vs Republic documents by the applicants leaving its prayers that said
FACTS: A petition for review assailing the Court of Appeals Sept application be denied and land in question be reverted to the
ownership of the Republic of the Philippines. At the appellate
19 Tacder NatRes

court, the RTC’s decision favoring the registrations of the land “Unless public land is shown to have been reclassified or
applied were reversed and thus this petition. alienated to a private person by the State, it remains part of the
inalienable public domain. Indeed, occupation thereof in the
ISSUE: WON the CA’s did not err in its decision to reverse the concept of owner, no matter how long, cannot ripen into
trial court findings. ownership and be registered as a title.

HELD: No. The petition is without merit.

The Court cited Sec. 48 of Commonwealth Act No. 141 as


amended, provide the registration the registration of the title of
lands in this wise; Homestead Settlement

“(b) Who by themselves, or their predecessors in interest have Mesina vs Sonza


been in open, continuous, exclusive, and notorious occupation of FACTS: Plaintiff Mesina claimed to be the owner of a parcel of
the land in the concept of the owner.” land located in San Antonio, Nueva, Ecija. He has been in
possession of the subject land openly, publicly and peacefully
The Court thus observed that the petitioners have insufficient since 1914. The said lot was subject of registration proceedings.
evidence on this.” Surprisingly, the defendant Pineda without knowledge of the
Plaintiff was able to procure a homestead patent in the same
The Court likewise pointed that; court were the registration of property was pending of the same
 Applicants to avail the OCT they must overcome the land by the plaintiff, despite of the fact that defendant had not
presumption that the land they applied for forms part of complied with the requirements of CA 141. That the said title was
the public domain and if so, it has to be shown as re- procured by defendants through frauds, deception and
classified or alienated to a private person by the state. misrepresentation since they knew that the lot belong to the
 Otherwise, the land remain inalienable public domain. plaintiff. Thus, Plaintiff sought to annulled and cancelled the
 To overcome this, an incontrovertible or clear and patent issued to defendant and prayed that this registration case
convincing evidence must be presented. pending in the same court be given due course.
 The court observed that applicants possession were
tacked only as far back as 1948 to its predecessors of ISSUE: WON the homestead patent given to defendant Pineda be
interest –the same must have shown that their declared null and void
predecessors in interest were in possession of the
property by some 30-years back or 1938 before WW-2. RULING: In view of the fact that plaintiff was able to proved his
open, continuous, exclusive possession of the disputed land for
The court finds the petitioners failed to show it even as they more than thirty years or since 1914 and that lot is at present
claimed that Cirilo Menguito once declared the land for tax subject of registration proceeding. Plaintiff is deemed to have
purposes in 1943 –yet failed to show documentary evidence to acquired the lot by grant of the state, it follows that the same
support the claim. had ceased to part of the public and had become private
The court therefore find no reason to modify the appellate court’s property and therefore beyond the control of the Director of
decision -thus denied the petition and declared cost against Land. The homestead patent issued to defendant therefore is null
petitioner. and void and for having it issued through fraud, deceit and
misrepresentation. The case was remanded to the trial court for
further proceedings.
Note:
20 Tacder NatRes

 After making the proper administrative investigation, the


Director of Lands overruled the opposition of Valentin Susi
and sold the land to Angela Razon and issued the proper
Susi vs Razon certificate of title to Angela Razon.
Facts:  Angela Razon required Valentin Susi to vacate the land in
 A complaint filed by Valentin Susi against Angela Razon question.
and the Director of Lands, praying for judgment: (a) Issue:
Declaring plaintiff the sole and absolute owner of the  Who is then the rightful owner of the land?
parcel of land; (b) annulling the sale made by the Director Held:
of Lands in favor of Angela Razon, on the ground that the  SC in their decision favoured Valentin Susi. According to
land is a private property; (c) ordering the cancellation of SC there is, the presumption juris et de jure established in
the certificate of title issued to said Angela Razon. paragraph (b) of section 45 of Act No. 2874, amending Act
 The Director of Lands denied each and every allegation No. 926, that all the necessary requirements for a grant
contained therein and, as special defense, alleged that by the Government were complied with, for he has been
the land in question was a property of the Government of in actual and physical possession, personally and through
the United States under the administration and control of his predecessors, of an agricultural land of the public
the Philippine Islands before its sale to Angela Razon, domain openly, continuously, exclusively and publicly
which was made in accordance with law. since July 26, 1894, with a right to a certificate of title to
 The Court of First Instance of Pampanga rendered said land under the provisions of Chapter VIII of said Act.
judgment declaring the plaintiff entitled to the possession So that when Angela Razon applied for the grant in her
of the land, annulling the sale made by the Director of favor, Valentin Susi had already acquired, by operation of
Lands in favor of Angela Razon, and ordering the law, not only a right to a grant, but a grant of the
cancellation of the certificate of title issued to her. Government, for it is not necessary that certificate of title
 The evidence shows that on December 18, 1880, Nemesio should be issued in order that said grant may be
Pinlac sold the land in question, then a fish pond, to sanctioned by the courts, an application therefore is
Apolonio Garcia and Basilio Mendoza. After having been in sufficient, under the provisions of section 47 of Act No.
possession thereof for about eight years, and the fish 2874. If by a legal fiction, Valentin Susi had acquired the
pond having been destroyed, Apolonio Garcia and Basilio land in question by a grant of the State, it had already
Mendoza sold it to Valentin Susi. ceased to be the public domain and had become private
 Before the execution of the deed of sale, Valentin Susi had property, at least by presumption, of Valentin Susi,
already paid its price and sown "bacawan" on said land, beyond the control of the Director of Lands. Consequently,
availing himself of the firewood gathered thereon, with in selling the land in question to Angela Razon, the
the proceeds of the sale of which he had paid the price of Director of Lands disposed of a land over which he had no
the property. The possession and occupation of the land in longer any title or control, and the sale thus made was
question, first, by Apolonio Garcia and Basilio Mendoza, void and of no effect, and Angela Razon did not thereby
and then by Valentin Susi has been open, continuous, acquire any right.
adverse and public, without any interruption, except
during the revolution, or disturbance, except when Angela
Razon commenced an action in the Court of First Instance
of Pampanga to recover the possession of said land.
 Having failed in her attempt to obtain possession of the
land in question through the court, Angela Razon applied
to the Director of Lands for the purchase thereof.
21 Tacder NatRes

inherited the above-mentioned portion from their late father,


Eufrosino M. Alba, who purchased the same from Dionisia Regado
in 1918. Hence, they have been in possession continuously,
openly and peacefully under claim of ownership of the above-
Sale of Public Agricultural Land mentioned portion for not less 70 years.
Lease
Confirmation of Imperfect Title
The private respondent/applicant requested the Municipal
Assessor of Banga to issue a revised tax declaration covering
Alba vs CA 4,845 square meters on the bare claim that "the area has been
Facts: Applicant Jose Lachica filed this application for title to land decreased" to only 620 square meters.
on April 28, 1958 with the claim that the land applied for was
purchased by him and his wife, Adela Raz from one Eulalio Raz. Issue: Whether or not the private respondent/applicant is entitled
The documents attached to the application are: technical to the confirmation of his ownership in fee simple for the 4, 845
description, surveyor's certificate, certification by the chief square meter parcel of land he applied for.
deputy assessor of Aklan and the blue print of Psu-161277.
The land applied for is residential and initial hearing was held on Held: The trial court and the Court of Appeals confirmed private
October 31, 1958. An order of general default was issued but respondent/applicant's title to the land on the basis of the
those who presented their opposition, namely, Octabela Alba findings that:
Vda. De Raz, Manuel and Susana Braulio, Jose Rago, representing 1. the private respondent/applicant purchased the land from
Apolonia Rebeco, the Director of Lands and the Municipality of Faustino Martirez;
Banga represented by the Provincial Fiscal, were given thirty (30) 2. the subject land is covered by Tax Declaration No. 14181;
days to file their written opposition. 3. the private respondent/applicant has paid the realty taxes
Manuel C. Braulio and Susana P. Braulio filed their opposition on on the land from 1945 up to the filing of his application in
October 31, 1958. They opposed the registration of the 1958;
southeastern portion of the 240 square meters of the land 4. the private respondent/applicant has been in actual, open
applied for alleging that they are the owners in fee simple and and continuous possession of the subject land in the
possessors of said portion and all the improvements thereon for concept of owner since 1945, and
not less than 70 years together with their predecessor-in-interest 5. the private respondent/applicant has acquired the land by
deriving their title by purchase from the original owners. prescription.
Jose Rago filed his opposition on November 29, 1958 as the duly
constituted attorney-in-fact of Apolonia Rebeco although no Private respondent that the acquired land in question from three
special power of attorney was attached. He alleged that his (3) sources, namely:
principal is the owner by right of succession and is in the a) A Deed of Sale dated August 13, 1941 allegedly executed
possession of said portion with all its improvements for more by Faustino Martirez covering 840 square meters;
than 80 years together with his predecessor-in-interest, b) 300 square meters allegedly purchased from private
continuously, peacefully and openly under claim of ownership. respondent's father-in-law Eulalio Raz, and
Rodolfo Alba, Lourdes Alba, represented by their attorney-in-fact, c) 3,725 square meters private respondent allegedly bought
alleged that they are the co-owners of a portion of the land in 1940 from Eugrocino Alba.
applied for with an area of 2,262 square meters bounded on the
north by Januario Masigon, Nicolas Realtor, Agustina Rebeldia In Section 48 of Commonwealth Act 141, as amended by RA Nos.
and Apolonia Rebeco, on the south by Eulalio Raz and on the 1942 and 6236, 48 which states that:
west by the public market of Banga. They claimed to have Sec. 48. The following-described citizens of the Philippines,
22 Tacder NatRes

occupying lands of the public domain or claiming to own any lands; and, 2.] Inalienable or non-disposable public lands. Non-
such lands or an interest therein, but whose titles have not been disposable public lands or those not susceptible of private
perfected or completed, may apply to the Court of First Instance appropriation include a.] Timber lands; and, b.] Mineral lands. 53
of the province where the land is located for confirmation of their For purposes of administration and disposition, the lands of the
claim and issuance of a certificate of title therefor, under the public domain classified as "disposable" or "alienable" are further
Land Registration Act, to wit: sub-classified into a.] Agricultural; b.] Residential, commercial,
1. Those who prior to the transfer of sovereignty from Spain industrial or for similar productive purposes; c.] Educational,
to the United States have applied for the purchase, charitable or other similar purposes, and d.] Reservations for
composition or other form of grant of lands of the public town sites and for public and quasi-public purposes. 54
domain under the laws and royal decrees then in force
and have instituted and prosecuted the proceedings in Rule:
connection therewith, but have with or without default The Decision of the Regional Trial Court of Kalibo, Aklan, Branch 1
upon their part, or for any other cause, not received title dated August 18, 1992 in Land Registration Case No. K-101, LRC
therefor, if such applicants or grantees and their heirs Record No. K-15104 is hereby MODIFIED as follows:
have occupied and cultivated said lands continuously 1. The 620 square meter portion on which private
since the filing of their applications. 49 respondent Jose N. Lachica's house is situated, clearly
2. Those who by themselves or through their predecessors in delineating its metes and bounds, is hereby ORDERED
interest have been in open, continuous, exclusive and segregated from the parcel of land described in Psu-
notorious possession and occupation of agricultural lands 161277 situated in the Poblacion of the Municipality of
of the public domain under a bona fide claim of Banga, Province of Aklan, Philippines with an area of
ownership, for at least thirty years immediately preceding 4,484 square meters, to be registered and confirmed in
the filing of the application for confirmation of title except the name of private respondent;
when prevented by war or force majeure. These shall be 2. A ten (10) meter road width along the National road
conclusively presumed to have performed all the mentioned in the application be segregated for future
conditions essential to a Government grant and shall be road widening programs upon the payment of just
entitled to a certificate of title under the provisions of this compensation to be annotated at the back of the title.
chapter. 50 3. Insofar as the ownership of the remainder of the subject
3. Members of the national cultural minorities who by land is concerned, the case is hereby REMANDED to the
themselves or through their predecessors-in-interest have court of origin for the reception of further evidence for the
been in open, continuous, exclusive and notorious petitioners to establish the other requisites for the
possession and occupation of lands of the public domain confirmation of title and registration in their names of the
suitable to agriculture, whether disposable or not, under a areas they respectively claim.
bona fide claim of ownership for at least 30 years shall be
entitled to the rights granted in subsection (b) hereof. 51
A circumspect scrutiny of the assailed Decision readily shows Republic vs CA and Naguit
that in the affirming the ruling of the trial court, the Court of FACTS: Corazon Naguit filed a petition for registration of title
Appeals relied on the provisions of Section 19 of Act 496 52 in which seeks judicial confirmation of her imperfect title over a
relation to the Civil Code's provision's on prescription on the parcel of land in Nabas, Aklan. It was alleged that Naguit and her
assumption that the subject land is private land. Therein lies the predecessors-in-interest have occupied the land openly and in
flaw in the appellate court's postulate. The application for the concept of owner without any objection from any private
registration of private respondent is for judicial confirmation of an person or even the government until she filed her application for
imperfect title considering that the land is presumed under the registration. The MCTC rendered a decision confirming the title in
Regalian Doctrine to be part of the public domain. the name of Naguit upon failure of Rustico Angeles to appear
Public lands are broadly classified into 1.] Alienable or disposable
23 Tacder NatRes

during trial after filing his formal opposition to the petition. 12, 1945 would not be susceptible to original registration, no
matter the length of unchallenged possession by the occupant.
The Solicitor General, representing the Republic of the In effect, it precludes the government from enforcing the said
Philippines, filed a motion for reconsideration on the grounds that provision as it decides to reclassify lands as alienable and
the property which is in open, continuous and exclusive disposable.
possession must first be alienable. Naguit could not have
maintained a bona fide claim of ownership since the subject land The land in question was found to be cocal in nature, it having
was declared as alienable and disposable only on October 15, been planted with coconut trees now over fifty years old. The
1980. The alienable and disposable character of the land should inherent nature of the land but confirms its certification in 1980
have already been established since June 12, 1945 or earlier. as alienable, hence agricultural. There is no impediment to the
application of Section 14 (1) of the Property Registration Decree.
ISSUE: Naguit had the right to apply for registration owing to the
Whether or not it is necessary under Section 14 (1) of the continuous possession by her and her predecessors-in-interest of
Property Registration Decree that the subject land be first the land since 1945.
classified as alienable and disposable before the applicant’s
possession under a bona fide claim of ownership could even
start.
Heirs of Malabanan vs Republic
RULING: FACTS:
Section 14 (1) merely requires that the property sought to be
registered as already alienable and disposable at the time the On 20 February 1998, Mario Malabanan filed an application for
application for registration of title is filed. land registration before the RTC of Cavite-Tagaytay, covering a
parcel of land situated in Silang Cavite, consisting of 71,324
There are three requirements for registration of title, square meters. Malabanan claimed that he had purchased the
1. that the subject property is alienable and disposable; property from Eduardo Velazco, and that he and his
2. that the applicants and their predecessor-in-interest have predecessors-in-interest had been in open, notorious, and
been in open, continuous, and exclusive possession and continuous adverse and peaceful possession of the land for more
occupation, and; than thirty (30) years. Velazco testified that the property was
3. that the possession is under a bona fide claim of originally belonged to a twenty-two hectare property owned by
ownership since June 12, 1945. his great-grandfather, Lino Velazco. Lino had four sons–
Benedicto, Gregorio, Eduardo and Esteban–the fourth being
There must be a positive act of the government through a statute Aristedes’s grandfather. Upon Lino’s death, his four sons
or proclamation stating the intention of the State to abdicate its inherited the property and divided it among themselves. But by
exclusive prerogative over the property, thus, declaring the land 1966, Esteban’s wife, Magdalena, had become the administrator
as alienable and disposable. However, if there has been none, it of all the properties inherited by the Velazco sons from their
is presumed that the government is still reserving the right to father, Lino. After the death of Esteban and Magdalena, their son
utilize the property and the possession of the land no matter how Virgilio succeeded them in administering the properties, including
long would not ripen into ownership through acquisitive Lot 9864-A, which originally belonged to his uncle, Eduardo
prescription. Velazco. It was this property that was sold by Eduardo Velazco to
Malabanan.
To follow the Solicitor General’s argument in the construction of
Section 14 (1) would render the paragraph 1 of the said provision Among the evidence presented by Malabanan during trial was a
inoperative for it would mean that all lands of public domain Certification dated 11 June 2001, issued by the Community
which were not declared as alienable and disposable before June
24 Tacder NatRes

Environment & Natural Resources Office, Department of June 1945 and does not require that the lands should have
Environment and Natural Resources (CENRO-DENR), which stated been alienable and disposable during the entire period of
that the subject property was “verified to be within the Alienable possession, the possessor is entitled to secure judicial
or Disposable land per Land Classification Map No. 3013 confirmation of his title thereto as soon as it is declared
established under Project No. 20-A and approved as such under alienable and disposable, subject to the timeframe
FAO 4-1656 on March 15, 1982.” On 3 December 2002, the RTC imposed by Section 47 of the Public Land Act.
approved the application for registration. b) The right to register granted under Section 48(b) of the
Public Land Act is further confirmed by Section 14(1) of
The Republic interposed an appeal to the Court of Appeals, the Property Registration Decree.
arguing that Malabanan had failed to prove that the property
belonged to the alienable and disposable land of the public 2. For purposes of Section 14(2) of the Property Registration
domain, and that the RTC had erred in finding that he had been in Decree may a parcel of land classified as alienable and
possession of the property in the manner and for the length of disposable be deemed private land and therefore
time required by law for confirmation of imperfect title. On 23 susceptible to acquisition by prescription in accordance
February 2007, the Court of Appeals reversed the RTC ruling and with the Civil Code?
dismissed the appliocation of Malabanan.
In complying with Section 14(2) of the Property Registration
Decree, consider that under the Civil Code, prescription is
ISSUES: recognized as a mode of acquiring ownership of patrimonial
property. However, public domain lands become only patrimonial
property not only with a declaration that these are alienable or
1. In order that an alienable and disposable land of the disposable. There must also be an express government
public domain may be registered under Section 14(1) of manifestation that the property is already patrimonial or no
Presidential Decree No. 1529, otherwise known as the longer retained for public service or the development of national
Property Registration Decree, should the land be classified wealth, under Article 422 of the Civil Code. And only when the
as alienable and disposable as of June 12, 1945 or is it property has become patrimonial can the prescriptive period for
sufficient that such classification occur at any time prior the acquisition of property of the public dominion begin to run.
to the filing of the applicant for registration provided that a) Patrimonial property is private property of the
it is established that the applicant has been in open, government. The person acquires ownership of
continuous, exclusive and notorious possession of the patrimonial property by prescription under the Civil Code
land under a bona fide claim of ownership since June 12, is entitled to secure registration thereof under Section
1945 or earlier? 14(2) of the Property Registration Decree.

In connection with Section 14(1) of the Property Registration b) There are two kinds of prescription by which patrimonial
Decree, Section 48(b) of the Public Land Act recognizes and property may be acquired, one ordinary and other
confirms that “those who by themselves or through their extraordinary. Under ordinary acquisitive prescription, a
predecessors in interest have been in open, continuous, person acquires ownership of a patrimonial property
exclusive, and notorious possession and occupation of alienable through possession for at least ten (10) years, in good
and disposable lands of the public domain, under a bona fide faith and with just title. Under extraordinary acquisitive
claim of acquisition of ownership, since June 12, 1945” have prescription, a person’s uninterrupted adverse possession
acquired ownership of, and registrable title to, such lands based of patrimonial property for at least thirty (30) years,
on the length and quality of their possession. regardless of good faith or just title, ripens into ownership.
a) Since Section 48(b) merely requires possession since 12
25 Tacder NatRes

State. Land that is other than part of the patrimonial property of


3. May a parcel of land established as agricultural in the State, provinces, cities and municipalities is of private
character either because of its use or because its slope is ownership if it belongs to a private individual. Pursuant to the
below that of forest lands be registrable under Section Regalian Doctrine (Jura Regalia), all lands of the public domain
14(2) of the Property Registration Decree in relation to the belong to the State. All lands not appearing to be clearly under
provisions of the Civil Code on acquisitive prescription? private ownership are presumed to belong to the State. Also,
4. Are petitioners entitled to the registration of the subject public lands remain part of the inalienable land of the public
land in their names under Section 14(1) or Section 14(2) domain unless the State is shown to have reclassified or
of the Property Registration Decree or both? alienated them to private persons.

It is clear that the evidence of petitioners is insufficient to Classifications of public lands according to alienability
establish that Malabanan has acquired ownership over the 1935 Constitution: lands of the public domain were classified into
subject property under Section 48(b) of the Public Land Act. - agricultural, timber and mineral. Section 10, Article XIV of the
There is no substantive evidence to establish that Malabanan or 1973 Constitution: lands of the public domain were classified into
petitioners as his predecessors-in-interest have been in - agricultural, industrial or commercial, residential, resettlement,
possession of the property since 12 June 1945 or earlier. The mineral, timber or forest, and grazing land, with the reservation
earliest that petitioners can date back their possession, that the law might provide other classifications. 1987
according to their own evidence—the Tax Declarations they Constitution adopted the classification under the 1935
presented in particular—is to the year 1948. Thus, they cannot Constitution into agricultural, forest or timber, and mineral, but
avail themselves of registration under Section 14(1) of the added national parks. Under Section 2, Article XII of the 1987
Property Registration Decree. Constitution, only agricultural lands of the public domain may be
alienated; all other natural resources may not be. Alienable and
Neither can petitioners properly invoke Section 14(2) as basis for disposable lands of the State fall into two categories:
registration. While the subject property was declared as alienable a) patrimonial lands of the State, or those classified as lands
or disposable in 1982, there is no competent evidence that is no of private ownership under Article 425 of the Civil Code,
longer intended for public use service or for the development of without limitation; and
the national evidence, conformably with Article 422 of the Civil b) lands of the public domain, or the public lands as provided
Code. The classification of the subject property as alienable and by the Constitution, but with the limitation that the lands
disposable land of the public domain does not change its status must only be agricultural.
as property of the public dominion under Article 420(2) of the Consequently, lands classified as forest or timber, mineral, or
Civil Code. Thus, it is insusceptible to acquisition by prescription. national parks are not susceptible of alienation or disposition
unless they are reclassified as agricultural. A positive act of the
2013 Government is necessary to enable such reclassification, and the
Classifications of land according to ownership exclusive prerogative to classify public lands under existing laws
Land may be classified as either of public dominion or of private is vested in the Executive Department, not in the courts. If,
ownership. however, public land will be classified as neither agricultural,
It is of public dominion if it: forest or timber, mineral or national park, or when public land is
1. is intended for public use; or no longer intended for public service or for the development of
2. belongs to the State, without being for public use, and is the national wealth, thereby effectively removing the land from
intended for some public service or for the development the ambit of public dominion, a declaration of such conversion
of the national wealth. must be made in the form of a law duly enacted by Congress or
Land belonging to the State that is not of such character, or by a Presidential proclamation in cases where the President is
although of such character but no longer intended for public use duly authorized by law to that effect.
or for public service forms part of the patrimonial property of the
26 Tacder NatRes

Disposition of alienable public lands of the Public Land Act, the agricultural land subject
Section 11 of the Public Land Act (CA No. 141) provides the of the application needs only to be classified as
manner by which alienable and disposable lands of the public alienable and disposable as of the time of the
domain, i.e., agricultural lands, can be disposed of: application, provided the applicant’s possession
1. For homestead settlement; and occupation of the land dated back to June 12,
2. By sale; 1945, or earlier. Thereby, a conclusive presumption
3. By lease; and that the applicant has performed all the conditions
4. By confirmation of imperfect or incomplete titles; essential to a government grant arises, and the
a) By judicial legalization; or applicant becomes the owner of the land by virtue
b) By administrative legalization (free patent). of an imperfect or incomplete title. By legal fiction,
the land has already ceased to be part of the
public domain and has become private property.
The core of the controversy herein lies in the proper b) Lands of the public domain subsequently classified
interpretation of Section 11(4), in relation to Section 48(b) of the or declared as no longer intended for public use or
Public Land Act, which expressly requires possession by a Filipino for the development of national wealth are
citizen of the land since June 12, 1945, or earlier. Bearing in mind removed from the sphere of public dominion and
such limitations under the Public Land Act, the applicant must are considered converted into patrimonial lands or
satisfy the following requirements in order for his application to lands of private ownership that 3 may be alienated
come under Section 14(1) of the Property Registration Decree, to or disposed through any of the modes of acquiring
wit: ownership under the Civil Code.
1. The applicant, by himself or through his predecessor-in- If the mode of acquisition is prescription, whether ordinary or
interest, has been in possession and occupation of the extraordinary, proof that the land has been already converted to
property subject of the application; private ownership prior to the requisite acquisitive prescriptive
2. The possession and occupation must be open, continuous, period is a condition sine qua non in observance of the law
exclusive, and notorious; (Article 1113, Civil Code) that property of the State not
3. The possession and occupation must be under a bona fide patrimonial in character shall not be the object of prescription. In
claim of acquisition of ownership; the case at bar, the petitioners failed to present sufficient
4. The possession and occupation must have taken place evidence to establish that they and their predecessors-in-interest
since June 12, 1945, or earlier; and had been in possession of the land since June 12, 1945. Without
5. The property subject of the application must be an satisfying the requisite character and period of possession -
agricultural land of the public domain. possession and occupation that is open, continuous, exclusive,
In sum, these are the rules relative to the disposition of public and notorious since June 12, 1945, or earlier - the land cannot be
land or lands of the public domain, namely: considered ipso jure converted to private property even upon the
1. As a general rule and pursuant to the Regalian Doctrine, subsequent declaration of it as alienable and disposable.
all lands of the public domain belong to the State and are Prescription never began to run against the State, such that the
inalienable. Lands that are not clearly under private land has remained ineligible for registration under Section 14(1)
ownership are also presumed to belong to the State and, of the Property Registration Decree. Likewise, the land continues
therefore, may not be alienated or disposed; to be ineligible for land registration under Section 14(2) of the
2. Exceptions: Property Registration Decree unless Congress enacts a law or the
a) Agricultural lands of the public domain are President issues a proclamation declaring the land as no longer
rendered alienable and disposable through any of intended for public service or for the development of the national
the exclusive modes enumerated under Section 11 wealth.
of the Public Land Act. If the mode is judicial
confirmation of imperfect title under Section 48(b)
27 Tacder NatRes

Palomo vs CA  Whether or not tax receipt can be use as a proof of


DOCTRINE: 1) Law governing natural resources that forest land ownership over the claiming lands.
cannot be owned by private persons. It is not registrable and
possession thereof, no matter how lengthy, cannot convert it into
private property, less such lands are reclassified and considered RULING:
disposable and alienable. 2) Neither do the tax receipts which  There is no question that the lots here forming part of the
were presented in evidence prove ownership of the parcels of forest zone were not alienable lands of the public domain.
land inasmuch as the weight of authority is that tax declarations The adverse possession which may be the basis of a grant
are not conclusive proof of ownership in land registration cases. of title in confirmation of imperfect title cases applies only
FACTS: to alienable lands of the public domain. It is in the law
 Diego Palomo is the owner of 15 parcels of land covered governing natural resources that forest land cannot be
by Executive Order No. 40. On 1916, he ordered the owned by private persons. It is not registerable and
registration of these lands and donated the same to his possession thereof, no matter how lengthy, cannot
heirs, Ignacio and Carmen Palomo two months before his convert it into private property, unless such lands are
death in April 1937. reclassified and considered disposable and alienable.
 Averment that the aforesaid OCT were lost during the  Tax receipts which were presented in evidence prove
Japanese occupation, Ignacio Palomo filed a petition for ownership of the parcels of land inasmuch as the weight
reconstitution with the Court of First Instance of Albay on of authority is that tax declarations are not conclusive
May 1970. The Register of Deeds of Albay issued Transfer proof of ownership in land registration cases. As to the
Certificates of Title Nos. 3911, 3912, 3913 and 3914 forfeiture of improvements introduced by petitioners, the
sometime in October 1953. fact that the government failed to oppose the registration
 July 1954 President Ramon Magsaysay issued of the lots in question is no justification for petitioners to
Proclamation No. 47 converting the area embraced by plead good faith in introducing improvements on the lots.
Executive Order No. 40 into the "Tiwi Hot Spring National
Park," under the control, management, protection and
administration of the defunct Commission of Parks and
Wildlife, now a division of the Bureau of Forest Bracewell vs CA
Development. Facts: The controversy involves a total of nine thousand six
 Land was never released as alienable and disposable hundred fifty-seven (9,657) square meters of land located in Las
portion of the public domain and, therefore, is neither Piñas, Metro Manila. In 1908, Maria Cailles, married to James
susceptible to disposition under the provisions of the Bracewell, Sr., who acquired the said parcels of land from the
Public Land Law nor registerable under the Land Dalandan and Jimenez families of Las Piñas; after which
Registration Act. The Palomos, however, continued in corresponding Tax Declarations were issued in the name of Maria
possession of the property, paid real estate taxes thereon Cailles.
and introduced improvements by planting rice, bananas,
pandan and coconuts. On January 16, 1961, Maria Cailles sold the said parcels of land to
 April 8, 1971, petitioner Carmen de Buenaventura and her son, the petitioner, by virtue of a Deed of Sale which was
spouses Ignacio Palomo and Trinidad Pascual mortgaged duly annotated and registered with the Registry of Deeds of
the parcels of land to guarantee a loan of P200,000 from Pasig, Rizal. Tax Declarations were thereafter issued in the name
the Bank of the Philippine Islands. of petitioner, canceling the previous Tax Declarations issued to
ISSUES: Maria Cailles. On September 19, 1963, petitioner filed before the
 Whether or not forest land which is a public domain may then Court of First Instance of Pasig, Rizal an action for
be owned by private persons confirmation of imperfect title under Section 48 of
28 Tacder NatRes

Commonwealth Act No. 141. asserted right to ownership in land and charged with the
conservation of such patrimony. This same doctrine also states
The Director of Lands, represented by the Solicitor General, that all lands not otherwise appearing to be clearly within private
opposed petitioner's application on the grounds that neither he ownership are presumed to belong to the State (Director of Lands
nor his predecessors-in-interest possessed sufficient title to the vs. Intermediate Appellate Court, 219 SCRA 340).
subject land nor have they been in open, continuous, exclusive
and notorious possession and occupation of the same for at least Hence, the burden of proof in overcoming the presumption of
thirty (30) years prior to the application, and that the subject State ownership of lands of the public domain is on the person
land is part of the public domain. applying for registration. The applicant must show that the land
subject of the application is alienable or disposable. These
On May 3, 1989, the lower court issued an Order granting the petitioners failed to do.
application of petitioner. The Solicitor General promptly appealed The homestead patent was issued to petitioners' predecessor-in-
to respondent Court which, on June 29, 1992, reversed and set interest, the subject land belong to the inalienable and
aside the lower court's Order. It also denied petitioner's Motion undisposable portion of the public domain. Thus, any title issued
for Reconsideration in its Resolution of September 30, 1992. in their name by mistake or oversight is void ab initio because at
the time the homestead patent was issued to petitioners, as
Issues: successors-in-interest of the original patent applicant, the
a) Whether the failure of the petitioner to prosecute his Director of Lands was not then authorized to dispose of the same
action for an unreasonable length of time? because the area was not yet classified as disposable public land.
b) Whether the tax declarations attached to the complaint Consequently, the title issued to herein petitioners by the Bureau
do not constitute acquisition of the lands applied for? of Lands is void ab initio.
Held: Neither has petitioner shown proof that the subject Forestry
Administrative Order recognizes private or vested rights under
The controversy is simple. On one hand, petitioner asserts his which his case may fall. We only find on record the Indorsement
right of title to the subject land under Section 48 (b) of of the Bureau of Forest Development from which no indication of
Commonwealth Act No. 141, having by himself and through his such exemption may be gleaned.
predecessors-in-interest been in open, continuous, exclusive and Having found petitioner to have no cause of action for his
notorious possession and occupation of the subject parcels of application for confirmation of imperfect title, we see no need to
land, under a bona fide claim of acquisition or ownership, since discuss the other errors raised in this petition.
1908. On the other hand, it is the respondents' position that
since the subject parcels of land were only classified as alienable Republic vs CA and Bernabe
or disposable on March 27, 1972, petitioner did not have any title FACTS:
to confirm when he filed his application in 1963. Neither was the Lot No. 622 of the Mariveles Cadastre was declared public land in
requisite thirty years possession met. a decision rendered before the last war in Cadastral Case No. 19,
LRC Cadastral Record No. 1097. On July 6, 1965 such lot was
A similar situation in the case of Reyes v. Court of Appeals, where segregated from the forest zone and released and certified by
a homestead patent issued to the petitioners' predecessor-in- the Bureau of Forestry (BOF) as an agricultural Land for
interest was cancelled on the ground that at the time it was disposition under the Public Land Act. On April 26, 1967,
issued, the subject land was still part of the public domain. Respondents filed in the CFI of Bataan a petition to reopen
Cadastral Case No. 19 to perfect their rights and register their
In the said case, this Court ruled as follows — titles to said lots. They alleged that they acquired ownership and
possession of said parcels of land by purchase from the original
Under the Regalian doctrine, all lands of the public domain owners thereof, whose possession of the same including that of
belong to the State, and that the State is the source of any
29 Tacder NatRes

the herein respondents, has always been continuous, open, and 20 days of possession and occupation of the lots involved,
active, exclusive, public, adverse and in the concept of owners counted from July 6, 1965 when the lots involved had been
for more than 30 years. The Director of Forestry filed an segregated from the forest zone and released by the BOF as an
opposition to the above petition but later withdrew the same agricultural land for disposition under the Public Land Act. As
upon verification of findings that this portion of the timberland such, respondents and their predecessors in interest could not
had already been released from the mass of the public forests. have possessed the lots for the required period of 30 years as
Subsequently, the Acting Prov. Fiscal of Bataan, for and in behalf disposable agricultural land.
of the Director of Lands filed his opposition alleging that the land
is still a Public Land and as such cannot be the subject of a land
registration proceeding under Act 496. The lower court Republic vs Bautista
adjudicated in favor or respondent Bernabes, finding that the FACTS:
latter have complied with all the terms and conditions entitling Apolonio Bautista Jr. acquired lot 17078 through succession,
them to a grant. This decision having become final, the when his father Apolonio Sr. died in 1987. He applied judicial
Commissioner of Land Registration issued the corresponding confirmation of imperfect title before MTC with testimonial
decrees of registration. On the other hand, petitioner DL through evidence that his father was in possession of the said lot since
the Solicitor Gen. filed a petition for review of the decrees. 1969 and acquired it from Jardin and Villanueva, through
Afterwards, he filed an Amended Petition for Review, adding: that notarized Deed of Absolute Sale dated February 15, 1971 and
respondents executed simulated deeds of sale conveying May 25, 1973 respectively. The MTC granted the application and
portions of the subject parcels to third parties for fictitious declared him as the owner in fee simple of the said land.
considerations in order to remove the same from the coverage of The Government of the Philippines appealed before CA,
Sec. 38 of Act 496, but in truth, buyers are mere dummies of contending that the testimony of Apolonio Jr. is an hearsay which
petitioners; hence, not purchasers for value. The Court of First should not be given probative value. The application of judicial
Instance denied this petition and on appeal, the CA affirmed the confirmation of imperfect title must comply with Sec. 48(b) of CA
questioned decision. Petitioner’s Motion for Reconsideration 141. The CA affirmed the ruling of MTC, it pointed out that the
having been denied for lack of merit; hence, this petition. Government of the Phil. did not timely object to the evidence
presented.

ISSUE: ISSUE: W/N the grant of judicial confirmation of imperfect title to


WON the lots claimed by respondents could be legally be the Apolonio Jr proper?
subject of a juridical confirmation of Title under Section 48 (b) of
Commonwealth Act 141 as amended by Republic Act 1942. RULING: The Supreme Court reversed the ruling of MTC and CA.
It held that the requisite period of possession must conform with
HELD: Sec 48(b) of Public Land Act as amended by RA 1942 which
No. The Supreme Court ruled that Sec. 48 (b) of CA 141, as provides that any person who applies for judicial confirmation, he
amended, applies exclusively to public lands. Forest lands or or his predecessor in interest must have been in open,
areas covered with forests are excluded. Thus, possession of continuous, exclusive and notorious possession and occupation
forest lands, however long cannot ripen into private ownership. of alienable and disposable land of public domain under the
A parcel of forest land is within the exclusive jurisdiction of the bonafide claim of ownership at least 30 years since June 12, 1945
Bureau of Forestry and beyond the power and jurisdiction of the or earlier.
cadastral court to register under the Torrens System. Thus, even
if the reopening of the cadastral proceedings was at all possible, In the present case, the lower court relied only on the testimony
private respondents have not qualified for a grant under Section of Bautista Jr. that his father acquired the land and in possession
48 (b) of CA 141. They can only be credited with 1 year, 9 mos. since 1969. He failed to prove that the transferor had in open,
30 Tacder NatRes

continuous, exclusive and notorious possession of said land for at the land and building are with MIAA still the ownership is with the
least 30 years since June 12, 1945 or earlier. Bautista Jr. has no Republic of the Philippines. MIAA also contends that it is an
personal knowledge of these facts. Lack of this evidence does not instrumentality of the government and as such exempted from
give the court the right to grant a judicial confirmation of real estate tax. That the land and buildings of MIAA are of public
imperfect title in favor of Bautista Jr. dominion therefore cannot be subjected to levy and auction sale.
On the other hand, the officers of Paranaque City claim that MIAA
is a government owned and controlled corporation therefore not
exempted to real estate tax.

Issues:
 Whether or not MIAA is an instrumentality of the
government and not a government owned and controlled
corporation and as such exempted from tax.
 Whether or not the land and buildings of MIAA are part of
the public dominion and thus cannot be the subject of
levy and auction sale.

Ruling:
Under the Local government code, government owned and
controlled corporations are not exempted from real estate tax.
MIAA is not a government owned and controlled corporation, for
to become one MIAA should either be a stock or non stock
corporation. MIAA is not a stock corporation for its capital is not
Non-registrable Land divided into shares. It is not a non stock corporation since it has
no members. MIAA is an instrumentality of the government
vested with corporate powers and government functions.
MIAA vs CA Under the civil code, property may either be under public
Facts: Manila International Airport Authority (MIAA) is the dominion or private ownership. Those under public dominion are
operator of the Ninoy International Airport located at Paranaque owned by the State and are utilized for public use, public service
City. The Officers of Paranaque City sent notices to MIAA due to and for the development of national wealth. The ports included in
real estate tax delinquency. MIAA then settled some of the the public dominion pertain either to seaports or airports. When
amount. When MIAA failed to settle the entire amount, the properties under public dominion cease to be for public use and
officers of Paranaque city threatened to levy and subject to service, they form part of the patrimonial property of the State.
auction the land and buildings of MIAA, which they did. MIAA
sought for a Temporary Restraining Order from the CA but failed The court held that the land and buildings of MIAA are part of the
to do so within the 60 days reglementary period, so the petition public dominion. Since the airport is devoted for public use, for
was dismissed. MIAA then sought for the TRO with the Supreme the domestic and international travel and transportation. Even if
Court a day before the public auction, MIAA was granted with the MIAA charge fees, this is for support of its operation and for
TRO but unfortunately the TRO was received by the Paranaque regulation and does not change the character of the land and
City officers 3 hours after the public auction. buildings of MIAA as part of the public dominion. As part of the
public dominion the land and buildings of MIAA are outside the
MIAA claims that although the charter provides that the title of commerce of man. To subject them to levy and public auction is
31 Tacder NatRes

contrary to public policy. Unless the President issues a ISSUE: Whether or not petitioner’s open, continuous, exclusive
proclamation withdrawing the airport land and buildings from and notorious possession and occupation of Lot 138 since 1894
public use, these properties remain to be of public dominion and and for many decades thereafter vests ipso jure or by operation
are inalienable. As long as the land and buildings are for public of law upon it a government grant, a vested title, to the subject
use the ownership is with the Republic of the Philippines. property.

HELD: No. There was no question that petitioner has been in


open, continuous, exclusive and notorious possession and
Roman Catholic vs Municipality of Buruanga occupation of Lot 138-B since 1894 as evidenced by the church
FACTS: Petitioner Roman Catholic Bishop of Kalibo was allegedly structure built thereon but there was no evidence to show that
the lawful owner and possessor of a parcel of residential and such possession and occupation extended to Lots 138-A and 138-
commercial land, designated as Lot 138. The Roman Catholic C beginning the same period. No single instance of the exercise
Church was built in 1984 in the middle portion of the said lot and by the petitioner of proprietary acts or acts of dominion over
has been in existence since then up to the present. That these lots was established. Its unsubstantiated claim that the
sometime in 1978, the Municipality of Buruanga constructed its construction of the municipal building as well as the subsequent
municipal building on the northeastern portion of the Lot 138 improvements thereon was by its tolerance does not constitute
after it obtained the permission of the then parish priest of proof of possession and occupation on the petitioner’s part.
Buruanga on the condition that the municipality remove all the Absent the important requisite of open, continuous, exclusive and
improvements it constructed thereon if and when the petitioner notorious possession and occupation thereon since 1894, no
needed the said lot. government grant or title to Lots 138-A and 138-C had vested
When the municipal building was razed by fire in 1989, upon the petitioner ipso jure or by operation of law.
petitioner, through its counsel requested the officials of the Possession is open when it is patent, visible, apparent, notorious
municipality to refrain from constructing its new building on the and not clandestine. It is continuous when uninterrupted,
same site because it is the property of the church and it needed unbroken and not intermittent or occasional; exclusive when the
the lot for its social projects. However, the construction of the adverse possessor can show exclusive dominion over the land
new municipal building on the same site proceeded. and an appropriation of it to his own use and benefit; and
Consequently, petitioner filed a complaint and prayed that it be notorious when it is so conspicuous that it is generally known and
declared the lawful owner and possessor of Lot 138. Petitioner talked of by the public or the people in the neighborhood. Use of
contends that it does not allegedly lose its possession or land is adverse when it is open and notorious.
ownership over the property if the possession or use by another
of the same is by mere tolerance.
In its answer, the municipality alleged that said lot was surveyed Amunategui vs Director of Forestry
as property of the municipality and that the said municipality Facts: There were two petitions for review on certiorari
alone had possessed the said land under the claim of title questioning the decision of the Court of Appeals which declared
exclusively for over fifty (50) years, exclusive of all other rights the disputed property as forest land, not subject to titling in favor
and adverse to all other claimants. of private persons, Borre and Amunategui.
After due trial, the trial court declared petitioner as the lawful The Director of Forestry, through the Provincial Fiscal of Capiz,
owner and possessor of Lot 138-B and the Municipality of also filed an opposition to the application for registration of title
Buruanga as the lawful owner and possessor of Lots 138-A and claiming that the land was mangrove swamp which was still
138-C, the said lots being public plaza for public use. On appeal, classified as forest land and part of the public domain.
the CA affirmed the ownership of petitioner over Lot 138-B but Another oppositor, Emeterio Bereber filed his opposition insofar
declared Lots 138-A and 138-C as property of public dominion. as a portion of Lot No. 885 containing 117,956 square meters
was concerned and prayed that title to said portion be confirmed
32 Tacder NatRes

and registered in his name. ACQUIRED. — This Court ruled in the leading case of Director of
Forestry v. Muñoz (23 SCRA 1184) that possession of forest lands,
Issue: WON the lot in question can be subject of registration and no matter how long, cannot ripen into private ownership. And in
confirmation of title in the name of the private person. Republic v. Animas (56 SCRA 499), we granted the petition on the
ground that the ares covered by the patent and title was not
Held: The opposition of the Director of Forestry was disposable public land, it being a part of the forest zone and any
strengthened by the appellate court's finding that timber licenses patent and title to said area is void ab initio. It bears emphasizing
had to be issued to certain licensees and even Jose Amunategui that a positive act of Government is needed to declassify land
himself took the trouble to ask for a license to cut timber within which is classified as forest and to convert it into alienable or
the area. It was only sometime in 1950 that the property was disposable land for agricultural or other purposes.
converted into fishpond but only after a previous warning from
the District Forester that the same could not be done because it 3. ID.; ID.; CONFIRMATION, OF IMPERFECT TITLE CASES; BURDEN
was classified as "public forest”. OF PROVING THAT THE REQUIREMENTS OF THE LAW HAVE BEEN
A forested area classified as forest land of the public domain MET, RESTS ON THE APPLICANT. — In confirmation of imperfect
does not lose such classification simply because loggers or title cases, the applicant shoulders the burden of proving that he
settlers may have stripped it of its forest cover. "Forest lands" do meets the requirements of Section 48, Commonwealth Act No.
not have to be on mountains or in out of the way places. Swampy 141, as amended by Republic Act No. 1942. He must overcome
areas covered by mangrove trees, nipa palms, and other trees the presumption that the land he is applying for is part of the
growing in brackish or sea water may also be classified as forest public domain but that he has an interest therein sufficient to
land. The possession of forest lands, no matter how long, cannot warrant registration in his name because of an imperfect title
ripen into private ownership. Therefore, the lot in question never such as those derived from old Spanish grants or that he has had
ceased to be classified as forest land of public domain. continuous, open, and notorious possession and occupation of
agricultural lands of the public domain under a bona fide claim of
1. CIVIL LAW; PUBLIC LAND ACT; FOREST LAND; CLASSIFICATION acquisition of ownership for at least thirty (30) years preceding
NOT LOST EVEN IF IT HAS BEEN STRIPPED OF FOREST COVER; the filing of his application.
UNLESS RELEASED IN AN OFFICIAL PROCLAMATION AS
DISPOSABLE LANDS, RULES ON CONFIRMATION OF IMPERFECT
TITLE DO NOT APPLY. — A forested area classified as forest land Ankron vs Government of the Philippines
of the public domain does not lose such classification simply FACTS: This case was commenced for the purpose of registering
because loggers or settlers may have stripped it of its forest a land under the Torrens system, which bounded, particularly
cover. Parcels of land classified as forest land may actually be situated in the plan description attached to the complaint and
covered with grass or planted to crops by kaingin cultivators or made part thereof. The respondent, Oppositor, The Government
other farmers. "Forest lands" do not have to be on mountains or of the Philippine islands alleged that the said land was the
in out of the way places. Swampy areas covered by mangrove property of the Government of the United States of America and
trees, nipa palms, and other tress growing in brackish or sea under the control of the Government of the Philippines. No proof
water may also be classified as forest land. The classification is was presented by the oppositor regarding its allegations. The
descriptive of its legal nature or status and does not have to be Judge render its decision favouring Akron, and ordered that the
descriptive of what the land actually looks like. Unless and until said land be registered to the latter, however that the right of the
the land classified as "forest" is released in an official government to open a road in a manner that the opening should
proclamation to that effect so that it may form part of the fifteen meters meters wide and should follow approximately the
disposable agricultural lands of the public domain, the rules on line of the road. Hence the Director of the lands appealed to this
confirmation of imperfect title do not apply. court. That all of said land, with the exception of a small part at
the north, the exact description and extension of which does not
2. ID.; ID.; FOREST LANDS; ACQUISITIVE OWNERSHIP NOT
33 Tacder NatRes

appear, has been cultivated and planted for more than forty-four timber or the discovery of valuable minerals, lands classified as
years prior to the date of this decision; agricultural today may be differently classified tomorrow. Each
That said land was formerly occupied, cultivated and planted by case must be decided upon the proof in that particular case,
Moros, Mansacas and others, under a claim of ownership, and having regard for its present or future value for one or the other
that they lived thereon and had their houses thereon, and that purposes. We believe, however, considering the fact that it is a
portion of the land which was not planted or cultivated was used matter of public knowledge that a majority of the lands in the
as pasture land whereon they pastured their carabaos, cattle, Philippine Islands are agricultural lands, that the courts have a
and horses; right to presume, in the absence of evidence to the contrary, that
in each case the lands are agricultural lands until the contrary is
That all of said Moros and Mansacas sold, transferred and shown. Whatever the land involved in a particular land
conveyed all their right, title and interest in said land to the registration case is forestry or mineral land must, therefore, be a
applicant, J. H. Ankron, some eleven years past, at which time all matter of proof. Its superior value for one purpose or the other is
of the said former owners moved o n to adjoining lands where a question of fact to be settled by the proof in each particular
they now reside; case. The fact that the land is a manglar [mangrove swamp] is
not sufficient for the courts to decide whether it is agricultural,
That the possession under claim of ownership of the applicant forestry, or mineral land. It may perchance belong to one or the
and his predecessors in interest was shown to have been open, other of said classes of land. The Government, in the first
notorious, actual, public and continuous for more than forty-four instance, under the provisions of Act No. 1148, may, by
years past, and that their claim was exclusive of any other right reservation, decide for itself what portions of public land shall be
adverse to all other claims; considered forestry land, unless private interests have intervened
before such reservation is made. In the latter case, whether the
That the applicant now has some one hundred fifty (150) hills of land is agricultural, forestry, or mineral, is a question of proof.
hemp, some eight thousand (8,000) cocoanut trees, a dwelling Until private interests have intervened, the Government, by
house, various laborers' quarters, store-building, large camarin virtue of the terms of said Act (No. 1148), may decide for itself
(storehouse of wood, a galvanized iron and other buildings and what portions of the "public domain" shall be set aside and
improvements on said land. reserved as forestry or mineral land.

Issue: WON the land in question cannot be registered? Issue: Whether or not the said land is owned by the government
of the Philippines.
Ruling: The mere fact that a tract of land has trees upon it or has
mineral within it is not of itself sufficient to declare that one is Ruling: Yes, The court held that the applicant proved and validly
forestry land and the other, mineral land. There must be some supplied the requisites for the registration of the said land into an
proof of the extent and present or future value of the forestry and agricultural land as per stated by paragraph 6 of section 54 of
of the minerals. While, as we have just said, many definitions Act No. 926. The important prerequisites for registration of land
have been given for "agriculture," "forestry," and "mineral" lands, imposed by said section 54, paragraph 6, are
and that in each case it is a question of fact, we think it is safe to a) that the land shall be agricultural public land as defined
say that in order to be forestry or mineral land the proof must by the Act of Congress of July 1, 1902;
show that it is more valuable for the forestry or the mineral which b) that the petitioner, by himself or his predecessors in
it contains than it is for agricultural purposes. (Sec. 7, Act No. interest, shall have been in the open, continuous,
1148.) It is not sufficient to show that there exists some trees exclusive and notorious possession and occupation of the
upon the land or that it bears some mineral. Land may be same under a bona fide claim of ownership for a period of
classified as forestry or mineral today, and, by reason of the ten years next preceding the taking effect of said Act. The
exhaustion of the timber or mineral, be classified as agricultural government failed to disrupt the said facts presented by
land tomorrow. And vice-versa, by reason of the rapid growth of the applicant.
34 Tacder NatRes

beneficiaries.
Hence, the court rendered its judgement in favor of the applicant
ISSUE:
1. Whether or not the property in question is covered by
Sta Rosa Dev’t vs CA CARP despite the fact that the entire property formed part
FACTS: The case is a petition regarding Department of Agrarian of a watershed area prior to the enactment of R.A No.
Reform Adjudication Board’s (DARAB) order of compulsory 6657
acquisition of petitioner’s property under the Comprehensive 2. Whether the petition of land conversion of the parcels of
Agrarian Reform Program (CARP). land may be granted?

Petitioner Sta. Rosa Development Corporation (SRRDC), was the HELD:


registered owner of two parcel of land situated at Brgy. Casile,
Cabuyao, Laguna. According to them, these lands are watersheds Watershed is one of those enumerated by CARP to be exempt
which provide clean and potable (drinkable) water to the from its coverage.
Canlubang community and that 90 light industries are located in  Art. 67 of PD 1067 provides that Any watershed or any
that area. area of land adjacent to any surface water or overlying
They were alleging respondents usurped its rights over their any ground water may be declared by the Department of
property thereby destroying the ecosystem. Since the said land Natural resources as a protected area.
provides water to the residents, respondents sought an  Watersheds may be defined as an area drained by a river
easement of a right of a way to and from Barangay Castile, to and its tributaries and enclosed by a boundary or divide
which, by counterclaim, Sta. Rosa sought ejectment against which separates it from adjacent watersheds.
respondents.
Respondents went to the DAR and filed a case for compulsory
We cannot ignore the fact that the disputed parcels of land form
acquisition of the Sta. Rosa Property under the Comprehensive
a vital part of an area that need to be protected for watershed
Agrarian Reform Program.
purposes. The protection of watersheds ensures an adequate
Compulsory acquisition is the power of the government to
supply of water for future generations and the control of
acquire private rights in land without the willing consent of its
flashfloods that not only damage property but cause loss of lives.
owner or occupant in order to benefit the society.
Protection of watersheds is an intergenerational responsibility
that needs to be answered now.
The said land was inspected by the Municipal and Agrarian
Reform Officer, and upon consensus of the authorities concerned,
Although evidence of petitioners is strong, the Supreme Court
they decided that the said land must be placed under
opines that the area must be maintained for watershed purposes
compulsory acquisition.
for ecological and environmental considerations despite the 88
families who are beneficiaries of the CARP. It is important that a
Petitioners filed an objection on the ground that: larger view of the situation be taken because of the thousands of
residents downstream if the watershed will not be protected and
 The area is not appropriate for agricultural purposes. maintained for its natural purpose.
 The area was rugged in terrain with slopes 18% and
above. (which falls under the exception in compulsory Despite Supreme Court’s strong opinion of protection of
acquisition of CARP) watersheds as an intergenerational responsibility, they, however
 The occupants of the land were illegal settlers or ordered to DARAB to conduct a re-evaluation of the case since
(squatters) who by no means are entitled to the land as the said land falls under exception.
35 Tacder NatRes

partly amended the Mortgage Law as well as the Law of the


Collado vs CA Indies. The Maura Law was the last Spanish land law promulgated
FACTS: Petitioner Edna T. Collado filed with the land registration in the Philippines. It required the "adjustment" or registration of
court an application for registration of a parcel of land (“Lot”), all agricultural lands, otherwise the lands would revert to the
situated in Antipolo Rizal. Attached to the application was a state.
technical description, stating “this survey is inside IN-12
Mariquina Watershed.” The Solicitor General filed oppositions to Four years later, Spain ceded to the government of the United
the application. Petitioners (Edna Collado and her co-applicants) States all rights, interests and claims over the national territory
allege that they have occupied the Lot since time immemorial. of the Philippine Islands through the Treaty of Paris of December
Their possession has been open, public, notorious and in the 10, 1898. In 1903, the United States colonial government,
concept of owners. They paid all real estate taxes and submitted through the Philippine Commission, passed Act No. 926, the first
evidence to prove that there have been 9 transfers of rights Public Land Act, which was described as follows:
among them and their predecessors-in-interest. RTC ruled in
favor of the petitioners for having presented sufficient evidence "Act No. 926, the first Public Land Act, was passed in pursuance
to establish registrable title over the property. of the provisions of the Philippine Bill of 1902. The law governed
the disposition of lands of the public domain. It prescribed rules
ISSUE: and regulations for the homesteading, selling and leasing of
portions of the public domain of the Philippine Islands, and
 WON petitioners have registrable title over the Lot. NO. prescribed the terms and conditions to enable persons to perfect
their titles to public lands in the Islands. It also provided for the
Petitioners concede that the Lot is inside the literal description of "issuance of patents to certain native settlers upon public lands,"
Marikina Watershed Reservation (MWR). Their main claim over for the establishment of town sites and sale of lots therein, for
the Lot is that “all Presidential proclamations like the the completion of imperfect titles, and for the cancellation or
proclamation setting aside the MWR are subject to private confirmation of Spanish concessions and grants in the Islands." In
rights.” EO 33 (which established the MWR) has a saving clause short, the Public Land Act operated on the assumption that title
that the reservations are “subject to existing private rights, if any to public lands in the Name: I. Concept of Jura Regalia Natural
there be.” Resources First Set_ Philippine Islands remained in the
government; and that the government’s title to public land
Under the Regalian Doctrine, all lands not otherwise appearing to sprung from the Treaty of Paris and other subsequent treaties
be clearly within private ownership are presumed to belong to between Spain and the United States. The term "public land"
the State. The Spaniards first introduced the doctrine to the referred to all lands of the public domain whose title still
Philippines through the Laws of the Indies and the Royal Cedulas, remained in the government and are thrown open to private
specifically, Law 14, Title 12, Book 4 of the Novisima Recopilacion appropriation and settlement, and excluded the patrimonial
de Leyes de las Indias which laid the foundation that "all lands property of the government and the friar lands."
that were not acquired from the Government, either by purchase
or by grant, belong to the public domain." Upon the Spanish Thus, it is plain error for petitioners to argue that under the
conquest of the Philippines, ownership of all "lands, territories Philippine Bill of 1902 and Public Land Act No. 926, mere
and possessions" in the Philippines passed to the Spanish Crown. possession by private individuals of lands creates the legal
presumption that the lands are alienable and disposable.
The Laws of the Indies were followed by the Ley Hipotecaria or
the Mortgage Law of 1893. The Spanish Mortgage Law provided Both the 1935 and 1973 Constitutions prohibited the alienation
for the systematic registration of titles and deeds as well as of all natural resources except agricultural lands of the public
possessory claims. The Royal Decree of 1894 or the "Maura Law" domain. The 1987 Constitution readopted this policy. Indeed, all
lands of the public domain as well as all natural resources
36 Tacder NatRes

enumerated in the Philippine Constitution belong to the State. mangrove swamps located in the municipality of Sapian, Capiz.
Ruperto Villareal applied for its registration on January 25, 1949,
Watershed Reservation is a Natural Resource: The term "natural alleging that he and his predecessors-in-interest had been in
resource" includes "not only timber, gas, oil coal, minerals, lakes, possession of the land for more than forty years. He was opposed
and submerged lands, but also, features which supply a human by several persons, including the petitioner on behalf of the
need and contribute to the health, welfare, and benefit of a Republic of the Philippines. After trial, the application was
community, and are essential to the well-being thereof and approved by the Court of First Instance of Capiz. The decision
proper enjoyment of property devoted to park and recreational was affirmed by the Court of Appeals. The Director of Forestry
purposes." then came to this Court in a petition for review on certiorari
claiming that the land in dispute was forestal in nature and not
 Did petitioners acquire private rights over the parcel of subject to private appropriation. He asks that the registration be
land prior to the issuance of EO 33? NO. reversed. It is undisputed by the parties that the land in dispute
is a mangrove land HOWEVER the legal nature of mangrove
An applicant must overcome the presumption that the land he is swamps or manglares are still in contention. Director of Forestry
applying for is part of the public domain and that he has an claims that it is forestall and is not disposable. On the other
interest to warrant registration in his name arising from an hand, Private respondents insists that it is alienable as
imperfect title (may have been derived from old Spanish grants agricultural land.
or titles). In the case at bar, petitioners were unable to acquire a
valid and enforceable right or title because of the failure to ISSUES: Are mangrove swamps classified as public forest lands?
complete the required period of possession (at least 30 years).
RULING: YES. Part of our public forest lands, they are not
Assuming that the Lot was alienable and disposable land prior to alienable under the Constitution or are they considered public
the issuance of EO 33 in 1904, EO 33 reserved the Lot as a agricultural lands; they may be acquired under private
watershed. Since then, the Lot became non-disposable and ownership.
inalienable public land. At the time petitioners filed their
application on April 25, 1985, the Lot has been reserved as a Mangrove swamps or manglares should be understood as
watershed under EO 33 for 81 years prior to the filing of comprised within the public forests of the Philippines as defined
petitioners’ application. in the aforecited Section 1820 of the Administrative Code of
1917. The legislature having so determined, we have no
NOTES: 2. Republic vs Dela Rosa (160 SCRA 228, G.R. No. L- authority to ignore or modify its decision, and in effect veto it, in
43938, April 15, 1988) FACTS: The case is about a parcel of land the exercise of our own discretion. The statutory definition
whose ownership is disputed by four parties: the dela Rosas, remains unchanged to date and, no less noteworthy, is accepted
Benguet Consolidated Inc (BCI), Atok Corp, and the Bureau of and invoked by the executive department. More importantly, the
Forestry Development (BFD). In 1965, Jose de la Rosa on his and said provision has not been challenged as arbitrary or unrealistic
on his three children’s behalf, applied to register a parcel of land or unconstitutional assuming the requisite conditions, to justify
divided into 9 lots in Benguet. According to the children, they our judicial intervention and scrutiny. The law is thus presumed
acquired the land by virtue of prescription. As evidence they valid and so must be respected. We repeat our statement in
produced tax declarations and realty tax receipts. the Amunategui case that the classification of mangrove swamps
as forest lands is descriptive of its legal nature or status and does
not have to be descriptive of what the land actually looks like.
That determination having been made and no cogent argument
Director of Forestry vs Villareal having been raised to annul it, we have no duty as judges but to
FACTS: The said land consists of 178,113 square meters of apply it.
37 Tacder NatRes

mineral claims owned by Atok has been declared under Tax


It follows from all this that the land under contention being Declaration No. 9535 and that in view of Presidential Decree No.
admittedly a part of the mangrove swamps of Sapian, and for 1214 an application for lease was filed by Atok covering the
which a minor forest license had in fact been issued by the Fredia mineral claim.
Bureau of Forestry from 1920 to 1950, it must be considered
forest land. It could therefore not be the subject of the adverse On the other hand, private respondent Liwan Consi has a lot
possession and consequent ownership claimed by the private below the land of a certain Mr. Acay at Tuding Slide, Itogon,
respondent in support of his application for registration. To be so, Benguet. He constructed a house thereon sometime in 1964. The
it had first to be released as forest land and reclassified as lot is covered by Tax Declaration No. 9462. When he first
agricultural land pursuant to the certification the Director of constructed his house below the lot of Mr. Acay he was told that
Forestry may issue under Section 1827 of the Revised it was not necessary for him to obtain a building permit as it was
Administrative Code. only a nipa hut. And no one prohibited him from entering the land
so he was constructing a house thereon. It was only in January
The Respondent even showed, a survey of the land and its tax 1984 when private respondent Consi repaired the said house that
declaration to support its claim, however the court held that the people came to take pictures and told him that the lot belongs to
same is insufficient especially now that the land is a forest land. Atok. Private respondent Consi has been paying taxes on said
land which his father before him had occupied .
WHEREFORE, the decision of the Court of Appeals is SET ASIDE
and the application for registration of title of private respondent On January 1984, the security guards of Atok informed Feliciano
is DISMISSED, with cost against him. This decision is immediately Reyes, Security Officer of Atok, that a construction was being
executory. undertaken at the area of the Fredia mineral claim by private
respondent Liwan Consi. Feliciano Reyes instructed the cashier to
go and take pictures of the construction. Feliciano Reyes himself
Atok-Big Wedge Mining Corp vs CA and other security guards went to the place of the construction
FACTS: Fredia Mineral claim of about nine (9) hectares situated in to verify and then to the police to report the matter.
Tuding, Itogon, Benguet, was located sometime between
December 25, 1930 and December 31, 1930, a period of six (6) On March 1, 1984, Atok filed a complaint for forcible entry and
days, by A.I. Reynolds in accordance with the provisions of the detainer against Liwan Consi , which was dismissed after due
Act of Congress of July 1, 1902, better known as the Philippine hearing by the MTC of Itogon in favor of Liwan Consi. Petitioner
Bill of 1902, in a so-called Declaration of Location. ATOK appealed to the RTC of Baguio, which reversed the decision
of the MTC, ordering defendant Liwan Consi to vacate the
The said Declaration of Location of mineral claim was duly premises of the Fredia Mineral claim, restoring possession thereof
recorded in the Office of the Mining Recorder sometime on to the plaintiff Atok Big Wedge Mining Company. Defendant Liwan
January 2, 1931. Fredia mineral claim, together with other Cosi was further ordered to remove and demolish the house he
mineral claims, was sold by A.I. Reynolds to Big Wedge Mining constructed in the premises of the land of Fredia Mineral claim.
Company, the earlier corporate name of Atok Big Wedge Mining
Company, Inc. (Atok for short; herein petitioner) in a Deed of Sale In a petition for review filed by Liwan Consi with the CA, the CA
executed on November 2, 1931. Since then petitioner Atok has rendered its decision dismissing the subject forcible entry action,
been in continuous and exclusive ownership and possession of and further rule in part that: Liwan Consi had a possessory right
said claim up to the present. over the property which may mature into ownership on the basis
of long-term possession under the Public Land Law. Thus, it held
Atok has paid the realty taxes and occupation fees for the Fredia that both Consi and ATOK are of equal footing with regards to the
mineral claim. The Fredia mineral claim together with other subject lot, holding possessory titles to the land. The petitioner
38 Tacder NatRes

through its long term occupancy while respondent mining firm of 1935 prohibited the alienation of all lands of the public domain
being the claim locator and applicant for lease on the mineral except agricultural lands, subject to vested rights existing at the
claim. time of its adoption. The land was not and could not have been
transferred to the private respondents by virtue of acquisitive
ATOK filed a motion for reconsideration, which was denied by the prescription, nor could its use be shared simultaneously by them
CA. Hence, this petition. and the mining companies for agricultural and mineral purposes
(Ibid).
ISSUE:
Whether or not an individual's long term occupation of land of On the matter of possession, private respondent contends that
the public domain vests him with such rights over the same as to his predecessor-in-interest has been in possession of said lot
defeat the rights of the owner of that claim. even before the war and has in fact cultivated the same. Since
the subject lot is mineral land, private respondent's possession of
HELD: It is of no importance whether Benguet and Atok had the subject lot no matter how long did not confer upon him
secured a patent for as held in the Gold Creek Mining Corporation possessory rights over the same.
case, for all physical purposes of ownership, the owner is not
required to secure a patent as long as he complies with the Furthermore, Article 538 of the New Civil Code provides:
provisions of the mining laws; his possessory right, for all
practical purposes of ownership, is as good as though secured by Art. 538. Possession as a fact cannot be recognized at the
patent (Republic v. Court of Appeals, 160 SCRA 228 [1988]). same time in two different personalities except in the cases of
co-possession. Should a question arise regarding the fact of
In the case at bar, the evidence on record pointed that the possession, the present possessor shall be preferred; if there are
petitioner Atok has faithfully complied with all the requirements two possessors, the one longer in possession; if the dates of the
of the law regarding the maintenance of the said Fredia Mineral possession are the same, the one who presents a title; and if all
Claim. these conditions are equal, the thing shall be placed in judicial
deposit pending determination of its possession or ownership
The perfection of the mining claim converted the property to through proper proceedings.
mineral land and under the laws then in force removed it from
the public domain. By such act, the locators acquired exclusive Since 1931 up to the present, petitioner ATOK has been in
rights over the land, against even the government, without need continuous and exclusive possession of the Fredia mineral claim
of any further act such as the purchase of the land or obtaining while private respondent's possession started only sometime in
of a patent over it. As the land had become the private property 1964 when he constructed a house thereon. Clearly, ATOK has
of the locators, they had the right to transfer the same, as they superior possessory rights than private respondent, Liwan Consi,
did, to Benguet and Atok . the former being "the one longer in possession."

As in the instant petition, the record shows that the lot in It is therefore clear that from the legal viewpoint it was really
question was acquired through a Deed of Sale executed between petitioner who was in actual physical possession of the property.
Atok and Fredia Mineral Claim. Having been deprived of this possession by the private
respondent, petitioner has every right to sue for ejectment.

It is, therefore, evident that Benguet and Atok have exclusive With this ruling enunciated by the Court, it can further be
rights to the property in question by virtue of their respective declared and held that petitioner Atok has the exclusive right to
mining claims which they validly acquired before the Constitution the property in question.
39 Tacder NatRes

On pre-trial the Republic, as plaintiff therein, marked (and later


offered in evidence)the Deed of Sale dated October 30, 1991 as
Republic vs Southside Homeowners its Exhibit "A,"and TCT No. 15084 as Exhibit "B."Respondent,
FACTS: The subject matter of these proceedings for declaration then defendant SHAI adopted Exhibits "A" and “B” as its Exhibits
of nullity of title are parcels of land with a total area of 39.99 "1" and “2,” respectively.
hectares, more or less, known as the JUSMAG housing area in
Fort Bonifacio where, military officers, both in the active and During the trial, the Republic presented as expert witness NBI
retired services, and their respective families, have been Document Examiner Eliodoro Constantino who testified on NBI
occupying housing units and facilities originally constructed by QDR No. 815-1093 and asserted that the signature of Palad in
the AFP. Exhibit “A” is a forgery. For his part, Palad dismissed as forged
his signature appearing in the same document and denied ever
Private respondent SHAI is a non-stock corporation organized signing the same, let alone in front of a notary public holding
mostly by wives of AFP military officers. Records show that SHAI office outside of the LMB premises. Pressing the point, Palad
was able to secure from the Registry of Deeds of the Province of stated that he could not have had signed the conveying deed
Rizal a title – Transfer Certificate of Title in its name to the bulk involving as it did a reservation area which, apart from its being
of, if not the entire, JUSMAG area. outside of the LMB’s jurisdiction, is inalienable in the first place.
The Rizal Registry issued TCT No. 15084 on October 30, 1991on
the basis of a notarized Deed of Sale purportedly executed on the For its part, then defendant SHAI presented an opposing expert
same date by then Director Abelardo G. Palad, Jr. of the Lands witness in the person of Police Inspector Redencion Caimbon who
Management Bureau (LMB) in favor of SHAI.The total purchase testified that Palad’s signature in Exhibit “A” is genuine. Mrs.
price as written in the conveying deed was P11,997,660.00 or Virginia Santos, then SHAI president, likewise testified, saying
P30.00 per square meter that applications to purchase were signed and then filed with the
It appears that in the process of the investigation conducted by LMB by one Engr. Eugenia Balis, followed by the payment in full
the Department of Justice on reported land scams at the FBMR, a of the contract price.
copy of the aforesaid October 30, 1991deed of sale surfaced and
eventually referred to the National Bureau of Investigation (NBI) Eventually, in a decision dated October 7, 1997, the trial court
for examination. The results of the examination undertaken by rendered judgment dismissing the Republic’s complaint as it
NBI Document Examiner Eliodoro Constantino reveals that the considered the parcels covered by the deed in question as no
puported signatures in the document are forgeries. longer part of the FBMR. Therefrom, the Republic went on appeal
to the CA which affirmed in toto that of the trial court.
On October 16, 1993, then President Fidel V.Ramos issued
Memorandum Order No. 173 directing the Office of the Solicitor Hence, this petition of the Republic.
General (OSG) to institute action towards the cancellation of TCT
No. 15084 and the title acquired by the Navy Officer’s Village ISSUE: Was the JUSMAG area, during the period material,
Association (NOVA) over a bigger parcel within the reservation. A alienable or inalienable, as the case may be, and, therefore, can
month later, the OSG, in behalf of the petitioner Republic, filed or cannot be subject of a lawful private conveyance?
with the RTC of Pasig City the corresponding nullification and
cancellation of title suit against the private respondent SHAI, RULING: Petitioner Republic, correctly asserts the inalienable
purported signature thereon of Palad is a forgery; b) there are no character of the JUSMAG area, the same having not effectively
records with the LMB of (i) the application to purchase and (ii) the been separated from the military reservation and declared as
alleged payment of the purchase price; and c) the property in alienable and disposable.
question is inalienable, being part of a military reservation
established under Proclamation No. 423. The President, upon the recommendation of the Secretary of
Environment and Natural Resources, may designate by
40 Tacder NatRes

proclamation any tract or tracts of land of the public domain as estopped by the mistake or error on the part of its officials or
reservations for the use of the Republic or any of its branches, or agents.
for quasi-public uses or purposes. Such tract or tracts of land
thus reserved shall be non-alienable and shall not be subject to Since the parcels of land in question allegedly sold to the private
sale or other disposition until again declared alienable. respondent are, or at least at the time of the supposed
Consistent with the foregoing postulates, jurisprudence teaches transaction were, still part of the FBMR, the purported sale is
that a military reservation, like the FBMR, or a part thereof is not necessarily void ab initio.
open to private appropriation or disposition and, therefore, not
registrable, unless it is in the meantime reclassified and declared Moreover, Article XII, Section 3 [of the 1987 Constitution forbids
as disposable and alienable public land. And until a given parcel private corporations from acquiring any kind of alienable land of
of land is released from its classification as part of the military the public domain, except through lease for a limited period.
reservation zone and reclassified by law or by presidential
proclamation as disposable and alienable, its status as part of a The interplay of compelling circumstances and inferences
military reservation remains, even if incidentally it is devoted for deducible from the case, also cast doubt on the authenticity of
a purpose other than as a military camp or for defense. The same such deed, if not support a conclusion that the deed is spurious.
is true in this case.
1. Palad categorically declared that his said signature on the
There is no doubt that the JUSMAG area subject of the questioned deed is a forgery. The NBI signature expert corroborated
October 30, 1991sale formed part of the FBMR as originally Palad’s allegation on forgery.Respondent SHAI’s expert
established under Proclamation No. 423. And while private witness from the PNP, however, disputes the NBI’s
respondent SHAI would categorically say that the petitioner findings. In net effect, both experts from the NBI and the
Republic had not presented evidence that “subject land is within PNP cancel each other out.
military reservation,” and even dared to state that the JUSMAG 2. Palad signed the supposed deed of sale in Manila,
area is the private property of the government and possibly at the LMB office at Plaza Cervantes, Binondo.
therefore removed from the concept of public domain per se its Even if he acted in an official capacity, Palad nonetheless
own evidence themselves belie its posture as their evidence both proceeded on the same day to Pasig City to appear
the TCT and the Deed of Sale technically described the property before the notarizing officer. The deed was then brought
as situated in Jusmag area located at Fort Bonifacio which is now to the Rizal Registry and there stamped “Received” by the
renamed Fort Mckinley a declared a military reservation. entry clerk. That same afternoon, or at 3:14 p.m. of
October 30, 1991to be precise, TCT No. 15084 was issued.
The Republic has, since the filing of its underlying complaint, In other words, the whole conveyance and registration
invoked Proclamation No. 423. In the process, it has invariably process was done in less than a day. The very unusual
invited attention to the proclamation’s specific area coverage to dispatch is quite surprising. Stranger still is why a bureau
prove the nullity of TCT No. 15084, inasmuch as the title head, while in the exercise of his functions as the bureau’s
embraced a reserved area considered inalienable, and hence, authorized contracting officer, has to repair to another
beyond the commerce of man. city just to have a deed notarized.
3. There is absolutely no record of the requisite public land
The October 30, 1991 Deed of Sale purportedly executed by application to purchase required under Section 89 of the
Palad, assuming its authenticity, could not plausibly be the Public Land Act. There is also no record of the deed of sale
requisite classifying medium converting the JUSMAG area into a and of documents usually accompanying an application to
disposable parcel. And private respondent SHAI’s unyielding purchase, inclusive of the investigation report and the
stance that would have the Republic in estoppel to question the property valuation. The Certification under the seal of the
transfer to it by the LMB Director of the JUSMAG area is LMB bearing date November 24, 1994 and issued/signed
unavailing. It should have realized that the Republic is not usually by Alberto Recalde, OIC, Records Management Division of
41 Tacder NatRes

the LMB pursuant to a subpoena issued by the trial court On 16 January 1956, by virtue of a final judgment in said case,
attest to this fact of absence of records. Atty. Alice B. supplemented byorders issued on March 21, 1956 and Aug. 13,
Dayrit, then Chief, Land Utilization and Disposition 1956, the Alagads were declared owners of Lot 1 and the
Division, LMB, testified having personally looked at the remaining portion, or Lot2, was declared public land. In August
bureau record book, but found no entry pertaining to 1966, the Alagads filed before the Municipal Court of Pila, Laguna
SHAI. an action to evict the barrio folk occupying portions of Lot 1. On 8
4. In its Answer as defendant a quo, respondent SHAI states August 1968, judgment was rendered in the eviction case
that the “deed of sale specifically meritorious Official ordering the barrio folktherein to return possession of the
Receipt No. 6030203 as evidence of full payment of the premises to the Alagads. The barrio folk did not appeal. The
agreed purchase price” An official receipt (O.R.) is Republic filed a petition for “annulment of title and reversion,
doubtless the best evidence to prove payment. While it insofar as the 1.42 hectare northwestern portion on end of Lot1 is
kept referring to O.R. No. 6030203 as its evidence of the concerned, contending that such is foreshore land, and that the
required payment, it failed to present and offer the receipt Alagads could not have had an imperfect title to it as it was the
in evidence. We can thus validly presume that no such OR barrio folk who filled up the land to elevate the land to its present
exists or, if it does, that its presentation would be adverse condition. The Court, issued a writ of preliminary injunction
to SHAI. enjoining the Provincial Sheriff of Laguna or his deputies from
A contract of sale is void where the price, which appears in the enforcing a writ of execution, and the Alagads from selling,
document as paid has, in fact, never been paid. mortgaging, disposing or otherwise entering into any transaction
affecting the area. The case was set for pre-trial on July6, 1971,
5. The purchase price was, according to the witnesses for to which the attorney representing the Republic did not appear.
SHAI, paid in full in cash to the cashier of the LMB the On July 16, 1971, the court dismissed the complaint. The
corresponding amount apparently coming in a mix of P500 Republic filed a motion for reconsideration, was set for hearing,
and P100 denominations. Albeit plausible, SHAI’s and finally denied by the court. Appeal was made to the Court of
witnesses’ account taxes credulity to the limit. Appeals, which sustained the trial court for failure to show in the
record on appeal that the appeal was perfected on time. Hence,
TCT No. 15084 of the Registry of Deeds of Rizal issued on the the appeal. The Supreme Court reversed the decision of the
basis of such Deed are declared void and cancelled lower courts, and reinstated the Republic’s complaint and thus
remanded the case to the trial court for further proceedings

Republic vs Alagad
Facts: On Oct. 11, 1951, Melitona, Carmen (with spouse State cannot be bound by or estopped from the mistakes
Espiridion Kolimlim), Justo, Carlos, Librada (with spouse Emerson or negligent acts of its officials or agents
Abano), Demetrio, and Antonio Alagad filed an application for The State cannot be bound by, or estopped from, the mistakes or
registration of their title over a parcel of land situated at Linga, negligent acts of its official or agents, much more, non-suited as
Pila, Laguna, with an area of 8.1263 hectares, which was a result thereof. This is so because the state as a persona in law
amended after the land was divided into two parcels, namely, Lot is the judicial entity, which is the source of any asserted right to
1 withan area of 5.2476 hectares and Lot 2 with an area of ownership in land under the basic doctrine embodied in the 1935
2.8421 hectares. The Republic opposed the application on the Constitution as well as the present charter. It is charged
stereo-typedground that applicants and their predecessors have moreover with the conservation of such patrimony. There is need
not been in possession of the land openly, continuously, publicly therefore of the most rigorous scrutiny before private claims to
and adversely under a bona fide claim of ownership since July 26, portions thereof are judicially accorded recognition. Such
1894 and the land has not ceased to be a part of the public primordial consideration, not the apparent carelessness, much
domain. It appears that barrio folk also opposed the application. less the acquiescence of public officials, is the controlling norm.
42 Tacder NatRes

Ramos v. Central Bank, and Nilo v. Romero not applicable banks, shores, roadsteads and others of similar character;
to the present case or if it
The cases of Ramos v. Central Bank of the Philippines and Nilo v. 2. belongs to the State, without being for public use, and are
Romero, are not applicable. In Ramos, the Court applied estoppel intended for some public service or for the development
upon finding of bad faith on the part of the State (the Central of the national wealth.
Bank) in deliberately reneging on its promises. In Nilo, the Court
denied efforts to impugn the jurisdiction of the court on the Patrimonial property and property of public dominion
ground that the defendant had been “erroneously” represented “All other property of the State which is not of the character
in the complaint by the City Attorney when it should have been mentioned in article [420], is patrimonial property,” meaning to
the City Mayor, on a holding that the City Attorney, in any event, say, property “open to disposition” by the Government, or
could have ably defended the City (Davao City). In both cases, it otherwise, property pertaining to the national domain, or public
is seen that the acts that gave rise to estoppel were voluntary lands. Property of the public dominion, on the other hand, refers
and intentional in character, in which cases, it could not be said to things held by the State by regalian right. They are things res
that the Government had been prejudiced by some negligent act publicae in nature and hence, incapable of private appropriation.
or omission. Thus, under the present Constitution, “[w]ith the exception of
agricultural lands, all other natural resources shall not be
Res judicata is not an impediment to reversion of alienated.”
property;
Republic v. CA, requisites for a prior judgment to become a bar Public Dominion, as to waters Article 502 provides that
Res judicata is not an impediment to reversion of property. In 1. Rivers and their natural beds;
Republic v. Court of Appeals, the Court stated that a certificate of 2. Continuous or intermittent waters of springs and brooks
title may be ordered cancelled (Republic v. Animus, et al.), and running in their natural beds and the beds themselves;
the cancellation may be pursued through an ordinary action 3. Waters rising continuously or intermittently on lands of
therefor. This action cannot be barred by the prior judgment of public dominion;
the land registration court, since the said court had no 4. Lakes and lagoons formed by Nature on public lands, and
jurisdiction over the subject matter. And if there was no such their beds;
jurisdiction, then the principle of res judicata does not apply. For 5. Rain waters running through ravines or sand beds, which
it is a well-settled rule that for a prior judgment to constitute a are also of public dominion;
bar to a subsequent case, the following requisites must concur; 6. Subterranean waters on public lands;
1. it must be a final judgment; 7. Waters found within the zone of operation of public
2. it must have been rendered by a court having jurisdiction works, even if constructed by a contractor;
over the subject matter and over the parties; 8. Waters rising continuously or intermittently on lands
3. it must be a judgment on the merits; and belonging to private persons, to the State, to a province,
4. there must be, between the first and second actions, or to a city or municipality from the moment they leave
identity of parties, identity of subject matter and identity such lands; and
of cause of action (Municipality of Daet vs. C4 93 SCRA 9. The waste waters of fountains, sewers and public
503; Mendoza vs. Arrieta, et al., 91 SCRA 113) establishments” are of public dominion.

Property of public dominion: It is also ordained in Article 44 of the Spanish Law of Waters of 3
Property for public use or public service “Property”, according to August 1866 that “natural ponds and lakes existing upon public
the Civil Code, “is either of public dominion or of private lands and fed by public waters, belong to the public domain.
ownership.” Property is of public dominion if it is Lakes, ponds, and pools existing upon the lands of private
1. intended for public use, such as roads, canals, rivers, individuals, or the State or provinces, belong to the respective
torrents, ports and bridges constructed by the State, owners of such lands, and those situated upon lands of
43 Tacder NatRes

communal use belong to their respective ‘pueblos.’” SIAN Enterprise vs FF Cruz


That the foreshore area had been reclaimed does not remove it
Laguna de Bay is a lake (Colegio de San Jose case); from its classification of foreshore area subject to the
Highest Ordinary Depth Laguna de Bay has long been recognized preferential right to lease of the littoral owner.
as a lake. Laguna de Bay is a body of water formed in FACTS: Western Visayas Industrial Corporation (WESVICO) filed a
depressions of the earth; it contains fresh water coming from foreshore lease application over the foreshore land adjacent to
rivers and brooks or springs, and is connected with Manila Bay by certain lots registered in its name. It eventually withdrew
the Pasig River. Inasmuch as Laguna de Bay is a lake, the Court the application and filed a petition for registration over the same
must resort to the legal provisions governing the ownership and foreshore land with the then Court of First Instance of Iloilo. The
use of lakes and their beds and shores, in order to determine the case was, however, archived as WESVICO‘s representative could
character and ownership of the parcels of land in question. The no longer be contacted, and later on, WESVICO has ceased
recourse to legal provisions is necessary, for under Article 74 of operations.
the Law of Waters, “the natural bed or basin of lakes is the
ground covered by their waters when at their highest ordinary F.F. Cruz & Co. (F.F. Cruz) filed with the Bureau of Lands, Iloilo City
depth” and in which case, it forms part of the national dominion. a foreshore lease application over a foreshore land, a portion of
When Laguna de Bay’s waters are at their highest ordinary depth which is adjacent to the lot previously occupied by WESVICO.
has been defined as the highest depth of the waters of Laguna Sian Enterprises Inc. (SIAIN) purchased the properties previously
de Bay during the dry season, such depth being the “regular, owned by WESVICO from the Development Bank of the
common, natural, which occurs always or most of the time during Philippines. It subsequently filed a foreshore
the year. Otherwise, where the rise in water level is due to the lease application over the foreshore land adjacent to the
“extraordinary” action of nature, rainfall for instance, the properties it bought from DBP.
portions inundated thereby are not considered part of the bed or
basin of the body of water in question. It cannot therefore be said Upon learning that 130 linear meters of the foreshore land
to be foreshore land but land outside of the public dominion, and subject of F.F. Cruz’s foreshore lease application overlapped that
land capable of registration as private property. covered by its foreshore lease application, SIAIN filed a protest 8
alleging that it being the owner of the property adjoining the
Foreshore land overlapping area, it should be given preference in its lease.
A foreshore land has been defined as “that part of (the land)
which is between high and low water and left dry by the flux and F.F. Cruz, argued that SIAIN must not be given preferential right
reflux of the tides,” or “The strip of land that lies between the since the area in dispute is classified as ―reclaimed‖ and that
high and low water marks and that is alternatively wet and dry the ownership was not by means of accretion. This argument has
according to the flow of the tide.” If the submergence, however, been sustained by the Land Management Bureau.
of the land is due to precipitation, it does not become foreshore,
despite its proximity to the waters. 9. Court not a trier of facts; Upon appeal to the DENR Secretary, SIAIN was upheld, declaring
not enough evidence to arrive a conclusive disposition; Remand that there was no basis to declare the area as ―reclaimed‖. F.F.
The case has to be decided alongside the above principles and Cruz however appealed to the Office of the President which
regretfully, the Court cannot make a ruling because it is not a overturned the decision of the DENR Secretary and found that
trier of facts, and it is in possession of no evidence to assist it in the area is reclaimed. On appeal, the Court of Appeals affirmed
arriving at a conclusive disposition. The Court thus remanded the the decision. Hence, the present petition. SIAIN contends that the
case to the court a quo to determine whether or not the property evidence overwhelmingly proves that the disputed area is
subject of controversy is foreshore. foreshore land and not reclaimed land which thus entitles it
preferential rights over the
44 Tacder NatRes

ISSUES: Whether the disputed land is a ―foreshore‖ or


Administrative Legalization (Free-Patent)
―reclaimed‖ area
- New RA 10023
Special Patents
HELD: That the foreshore area had been reclaimed does not
remove it from its classification of foreshore area subject to the
preferential right to lease of the littoral owner. Cadastral Registration Proceedings
It bears noting that it was not the reclamation a) Section 53 CA 141
that brought the disputed foreshore area into existence. Such b) Section 35-38 PD 1529
foreshore area existed even before F.F. Cruz undertook its
reclamation. It was ―formed by accretions or alluvial deposits
due to the action of the sea.‖ Following Santulan, the littoral
owner has preferential right to lease the same.
Director of Lands vs Roman Catholic
Contrary to the ruling of the Office of the President, as affirmed FACTS: In 1913, cadastral proceedings were conducted to settle
by the appellatecourt, littoral owner WESVICO cannot be the title to a considerable tract of land in the Province of Rizal.
considered to have waived or abandoned its preferential right to The Roman Catholic Archbishop of Manila (church) and other
lease the disputed area when it subsequently filed private parties were claimants of 13 cadastral lots that comprised
an application for registration thereover. For being a part of the the contested property. The lower court ruled in favor of the
public domain, ownership of the area could not be acquired by private claimants. Upon appeal, the church invoked that the
WESVICO. Its preferential right remained, however. Its move to composition title of the church with the Spanish Government
have the contested land titled in its name, albeit a faux pas, in included the subject property. The church then presented one
fact more than proves its interest to utilize it. witness and rested. The private oppositors then called their
respective witnesses to prove title by possession, and rested. The
As correctly argued by SIAIN, were WESVICO‘s petition for church thereafter made an offer to present additional testimony
registration which, as stated earlier, was archived by the trial intended to show that the possession of the private claimants
court, pursued but eventually denied, WESVICO would not have had been interrupted and that it was merely possession through
been barred from filing anew a foreshore lease application. the tolerance of the church. However, the counsel for the
Parenthetically, the petition for registration of WESVICO oppositors objected to the entry of additional testimonies which
was archived not on account of lack of interest but because it was sustained.
ceased operations due to financial reasons.
ISSUE: Did the lower court err in refusing the entry the church’s
additional testimonies?

RULING:
The Court ruled in the affirmative.
The object of a cadastral petition is that the title to the various
lots embraced in the survey may be settled and adjudicated. It is
in the nature of a proceeding in rem, promoted by the Director of
Lands, somewhat akin to a judicial inquiry and investigation
leading to a judicial decree. In one sense, there is no plaintiff and
there is no defendant. In another sense, the Government is the
plaintiff and all the- claimants are defendants.
45 Tacder NatRes

As a general rule, courts should adhere to the usual rules of dismissed on the ground of failure of proof of title in the plaintiff
practice, procedure, and evidence that governs registration at the time when the action was instituted and later when
proceedings. However, in registration proceedings where so judgment of dismissal was entered by the trial court.
many parties are involved and action is taken quickly and In July of 1918, or four months after the above-mentioned
abruptly, opportunity should be given to parties to submit decision of this Court, petitioner herein brought another action
additional corroborative evidence in support of their claims of for recovery of the land against the same defendants in the
title, if the ends of justice so require. This case was remanded previous case. The second suit was later dismissed by the Court
back to the lower court with the church being allowed to admit of First Instance and transferred to cadastral case No. 5 which
additional testimonies in the interest of justice and included the hacienda in question that had in the meantime been
ascertainment of truth. subdivided into lots. When the cadastral case came up before the
Hon. Meynardo M. Farol at Aringay, La Union, in July 1941, Fabian
Abellera vs Farol B.S. Abellera appeared as claimant while Narciso de Guzman and
Whether in a cadastral case, the judge may upon motion of others appeared as adverse claimant. The latter through counsel
adverse claimants order the cancellation of the claimant's moved that Abellera's claim over the lots concerned be dismissed
answer and keep the latter from introducing evidence to prove on the grounds of res judicata and prescription.
his ownership because the case is barred by a prior judgment, is A careful examination of the decision of this Court in the previous
the legal question at issue in this case. An order to that effect case (37 Phil., 865) convinces us that there is no res judicata. We
issued by the Court of First Instance of La Union, is impugned by merely held that Abellera had not acquired title to the hacienda
Fabian B.S. Abellera in a petition for a writ of certiorari. until the execution of the deed of acceptance and the notification
Abellera, in a previous case concerning the same real estate thereof, and we clearly refused to prevent Abellera from
involved herein, sued Hermegildo Balanag and others who are instituting a new action based upon his assertion that he had
either the same parties in this case or the latter's predecessors in acquired title to the estate since the dismissal of his original
interest, alleging ownership of the land. But his complaint was action.
dismissed by the Court of First Instance on two grounds: (1) The other ground for the motion for dismissal, prescription, is not
prescription in favor of defendants; and (2) the deed of donation involved in the present proceedings.
of these lands to him had not been formally accepted according The next question is: Did the cadastral court, on the ground of
to Article 633 of the Civil Code. Upon appeal to this Court, the res judicata, have any power to entertain the motion to dismiss
judgment of the trial court was affirmed on the second ground Abellera's claim and bar him from presenting evidence to prove
aforementioned (Abellera vs. Balanag G.R. No. 11970, his ownership of these lots?
promulgated March 22, 1918, and reported in 37 Phil. 865). Rule 132 of the Rules of Court provides:
It appears in that decision of this Court that after the perfection These rules shall not apply to land registration, cadastral and
of the appeal, Abellera executed a public document formally election cases, naturalization and insolvency proceedings, and
accepting the donation of the land, and presented and deed of other cases not herein provided for, except by analogy or in a
acceptance together with proofs of notification of acceptance to suppletory character and whenever practicable and convenient.
the donor, as ground for new trial. This Court held that this was The Rules of Court may be applied in cadastral cases when two
not newly-discovered evidence, and that Abellera had not conditions are present: (1) analogy or need to supplement the
acquired title to the hacienda until the execution of the deed of cadastral law, and (2) practicability and convenience.
acceptance and the notification thereof to donor. This Court If the nature and objective of the cadastral scheme are kept in
added: view, a motion to dismiss in a cadastral case on the ground of
So that whether rights he may have to institute and maintain a prior judgment would seem to be out of place. The Government
new action of ejectment in reliance upon his claim that he has initiates a cadastral case, compelling all claimants in a
acquired title to the hacienda, since the date of the dismissal of municipality to litigate against one another regarding their
this action, it is clear that the present action was properly respective claims of ownership. By this plan, all the private lands
in a town are registered in one single collective proceeding. Thus,
46 Tacder NatRes

the piece-meal and isolated registration of lands, so inadequate Gayapanao vs IAC


in more ways than one, is avoided. The principal aim is to settle FACTS: This is a petition for review on certiorari filed by Severino
as much as possible all disputes over land and to remove all Gayapanao and his siblings questioning the decision of the IAC in
clouds over land titles, as far a practicable, in a community. To upholding the validity of the sale of the land between their father
attain this purpose, the cadastral court should allow all claimants Constantino and their sister. The 2 hectare land, subject of this
ample freedom to ventilate whatever right they may assert over case is part of 10 hectare homestead land registered in the name
real estate, permitting them, in keeping with the law of evidence, of Constantino Gayapanao under OCT. The final order of the
to offer proofs in support of their allegations. To countenance the Director of Lands for the issuance of patent was issued on
contrary opinion, by suppressing the presentation of evidence in December 10, 1937. On November 15, 1938, Constantino
support of claims, would but serve to perpetuate conflicts over Gayapanao executed a private deed entitled kasulatan ng bilihan
land, for such stifled affirmations of ownership will fester like in favor of Simeona Gayapanao and his husband.
wounds unskillfully treated. No sufficient leeway having been
give all claimants to demonstrate the strength and consistently RTC’s decision: The contract of sale between Simeona
of their alleged rights, the stability of decrees of title is Gayapanao and her father is null and void for having been
jeopardized. executed with the 5 year prohibitory period provided under
In Haw Pia vs. Roman A. Cruz (G.R. No. 48506), we declared that Section 118 of the Public Land Law
the Court of First Instance in a cadastral proceeding cannot CA’s decision: It reversed the decision of the RTC and uphold the
appoint a receiver because its jurisdiction is special and limited. validity of the sale.
We declined in that case to apply the new Rules of Court by
analogy. ISSUE: WON CA was correct in upholding the validity of the sale?
We are, therefore, of the opinion that while in a cadastral
case res judicata is available to a claimant in order to defeat the RULING: No, the provision of law which prohibits the sale or
alleged rights of another claimant, nevertheless prior judgment encumbrance of the homestead within 5 years after the grant is
can not set up in a motion to dismiss. mandatory.
The order appealed from is hereby reversed. Petitioner herein
shall in the cadastral proceedings be allowed to present evidence From the date of the approval of the application and for a term of
to prove his claim over the lots in question. With costs against five (5) years from and after the date of issuance of the patent or
the adverse claimants who are respondents herein. So ordered. grant, lands acquired under free patent or homestead provisions
Yulo, C.J., Moran and Ozaeta, JJ. concur. cannot be subject to encumbrance or alienation, nor shall they
become liable to the satisfaction of any debt contracted prior to
the expiration of said period. The only exception mentioned by
the law is the sale or encumbrance in favor of the government or
any of its branches, units or institutions.
In a number of cases, we have consistently ruled that a sale of
homestead within the five (5) year prohibitive period is void ab
initio and the same cannot be ratified nor can it acquire validity
through the passage of time.

Prohibited Alienations and Transfer of Private Lands Republic vs Garcia


a) Sections 118-124 of CA 141 Sections 118-124
Sometime of CA
before the 141
last war appellant Isabelo Garcia and his
b) Section 7 Art XII, 1987 Constitution wife Tagumpay Dumaguindin acquired by purchase the
47 Tacder NatRes

homestead rights of Lingasa Bapanialag to a parcel of homestead [1]


void, and is a cause for reversion of the homestead to the
land situated at Mabay, Kiamba, Cotabato, containing an area of State.
23.21 hectares (Homestead Application No. 182259 [E-90722]).
The transfer was approved by the Secretary of Agriculture and Appellants' defense set up in the court below was that the
Natural Resources upon recommendation of the Director of document Exhibit A was intended merely as a mortgage on the
Lands and Patent No. V-532 was issued in favor of the appellant improvements and crops existing on the 19 hectares of the
Isabelo Garcia and his wife. Three years and three months after homestead land to secure the payment by instalment of a loan,
the issuance of the homestead patent, or on 14 April 1950, for only that it was drawn up by mistake as an absolute sale upon
and in consideration of the sum of P11,000, by an instrument the insistence of the vendees. If it was drawn up as an absolute
executed and acknowledged before a notary public the sale upon the insistence of the vendees, then there was no
appellant Isabelo Garcia and his wife sold and conveyed to mistake committed. The document Exhibit A is so clearly
Domingo Colorado, Raymundo de Guzman, Inocencio Padama, worded as to preclude an interpretation other than what the
Ignacio Ramos and Leon de Guzman 19 hectares of the parties had intended it to be a deed of absolute sale of the 19
homestead land (Exhibit A). The vendees took possession of hectares of the homestead land. Moreover, as found and held by
the part sold to them. The deed of sale was not submitted to the trial court:
the Secretary of Agriculture and Natural Resources for
approval nor presented to the Registrar of Deeds in and for the * * * as stated in Annex "A", the improvements on the land only
province of Cotabato for registration. consist of 50 coco,nut trees, 5 mango trees, 1 nangka tree,
bananas and other friut trees. It is unbelievable that
Section 118 of Commonwealth Act No. 141 partly provides: defendants Domingo Colorado, Raymundo de Guzman,
Except in favor of the Government or any of its branches, units, Inocencio Padama, Ignacio Ramos and Leon de Guzman would
or institutions, lands acquired under free patent or homestead grant a loan of P11,000.00 for such a small security as those
provision shall not be subject to encumbrance or alienation from improvements specified above. Then if it is true that the
the date of the approval of the application and for a term of five , transaction had between them is only a loan, to be paid by
years from and after the date of issuance of the patent or grant, instalment every year, the defendants failed to produce any
nor shall they become liable to the satisfaction of any debt evidence that any instalment has ever been paid, taking into
contracted prior to the expiration of said period, but the consideration that already elapsed more than five (5) years from
improvements or crops on the land may be mortgaged or that time up to the hearing of this case.
pledged to qualified persons, associations, or corporations. The fact that the appellant Isabelo Garcia moved to the
municipality of Bislig, province of Surigao, and abandoned his
Section 124 of the same Act provides: homestead in Kiamba, Cotabato, is proof that he sold 19
Any acquisition, conveyance, alienation, transfer, or other hectares of his homestead.
contract made or executed in violation of any of the provisions
of sections one hundred and eighteen, one hundred and twenty, Appellants contend that, under section 50, Act No. 496, the
one hundred and twenty-one, one hundred and twenty-two, and operative act to convey and affect lands registered thereunder is
one hundred and twenty-three of this Act shall be unlawful and the act of registration, that inasmuch as the deed of sale Exhibit
null and void from its execution and shall produce the effect of A was never registered there was actually no conveyance made
annulling and cancelling the grant, title, patent, or permit of the 19 hectares of the homestead land, and that for that
originally issued, recognized or confirmed, actually or reason there was no infringement of section 118 of
presumptively, and cause the reversion of the property and its Commonwealth Act No. 141. To constitute a violation of the
improvements to the State. section just referred to, it is enough that the homestead be
As the sale of the 19 hectares of the homestead land was made encumbered or alienated within the prohibitive period of five
within the prohibitive period of five years three years and three years; it is not necessary that the encumbrance or alienation be
months after the issuance of the homestead patent is null and registered in the Office of the Register of Deeds. To uphold the
48 Tacder NatRes

appellants' contention would defeat the very prohibition


established by law, for no party to a prohibited sale or HELD:
conveyance would register such ah illegal transaction. The Court of Appeals erroneously concluded that a prior tender
Besides, the vendees already, had taken possession of the part or offer of redemption is a prerequisite or precondition to the
sold to them. filing of the action for legal redemption. To avail of the right of
redemption what is essential is to make an offer to redeem within
Even if only 19 out of the 23.21 hectares of the homestead land the prescribed period. There is no prescribed form for an offer to
had been sold or alienated within the prohibitive period of five redeem to be properly effected. It can either be the formal tender
years from date of issuance of the patent to the grantee, such with consignation or the filing of a complaint in court. What is
alienation is a sufficient cause for reversion to the State of the paramount is the availment of the fixed and definite period within
whole grant. In granting a homestead to an applicant, the law which to exercise the right of legal redemption.
imposes as a condition that the land should not be encumbered,
sold or alienated within five years from the issuance of the The filing of the action itself is equivalent to a formal offer to
patent. The sale or alienation of part of the homestead violates redeem. What constitutes a condition precedent is either a
that condition. formal offer to redeem or the filing of an action in court together
The judgment appealed from is affirmed, with costs against with the consignation of the redemption price within the
the appellants. reglementary period.
The decision of respondent Court of Appeals is REVERSED and
SET ASIDE. The decision of the Regional Trial Court of Malolos,
Lee Chuy Realty Corp vs CA Bulacan is REINSTATED.
Facts: A valuable piece of land located at Meycauyan, Bulacan,
with an area of 24,576 sq. m. and covered by OCT No. 0-5290 is
disputed by Lee Chuy Realty Corporation and Marc Realty and
Development Corp.
On 5 May 1989 the Bascaras and Ernesto Jacinto also sold their
share to MARC REALTY which was registered on 16 October 1989.
Lee Chuy Realty claims that it was never informed of the
existence of the sale between Marc Realty and the
Bascaras/Jacinto. Marc Realty insists that Lee Chuy verbally
notified of the sale and was given a copy of the deed of sale.

On 13 November 1989 LEE CHUY REALTY filed a complaint for


legal redemption against MARC REALTY and consigned in court a Section 7 Art XII 1987 Constitution
manager's check for 614,400. MARC REALTY insisted that the
complaint be dismissed for failure to state a cause of action there Ong Ching Po vs CA
being no allegation of prior valid tender of payment or a prior FACTS:
valid notice of consignation.  Ong Joi Jong, sold a parcel of land located at Fundidor
Street, San Nicolas to private respondent Soledad Parian
on 23 July 1947. The sale was evidenced by a notarized
ISSUE: WON a formal offer to redeem accompanied with tender Deed of Sale, written in English, and was registered with
of payment a condition precedent to the filing of an action for the the Register of Deeds of Manila which in turn issued TCT
valid exercise of the right of legal redemption; is the filing of the no. 9260 dated 2 September 1947 in the name of private
action with consignation equivalent to a formal offer to redeem. respondent. According to private respondent, she
49 Tacder NatRes

entrusted the administration of the lot and building to Ong Frenzel vs Catito
Ching Po when she and her husband settled in Iloilo, DOCTRINE:
however when her husband died she demanded that the A contract that violates the Constitution and the law, is null and
lot be vacated since she was going to sell it, petitioners void and vests no rights and creates no obligations. It produces
refused to vacate said premises. On 19 March 1984, no legal effect at all. The petitioner, being a party to an illegal
Parian filed a case for unlawful detainer against Ong Ching contract, cannot come into a court of law and ask to have his
Po. The MTC dismissed her case and was reaffirmed by illegal objective carried out
both the RTC and CA.
FACTS: Petitioner Alfred Fritz Frenzel is an Australian citizen of
 On the other hand, on 6 December 1983, Ong Ching Po German descent. He was so enamored with Ederlina that he
executed a Deed of Absolute Sale conveying the same bought her numerous properties such as house and lot in Quezon
property to his children Jimmy and David Ong. On 12 City and in Davao City. He also put up a beauty parlor business in
December 1985, the Ong’s filed an action for the name of Ederlina. Alfred was unaware that Ederlina was
reconveyance and damages against Parian in RTC, Manila. married until her spouse Klaus Muller wrote a letter to Alfred
Upon the private respondent’s motion, this was begging the latter to leave her wife alone.
consolidated with her action for quieting of title against
the petitioners. The RTC rendered a decision in favor of When Alfred and Ederlinas relationship started deteriorating.
private respondent and was confirmed by the CA, hence Ederlina had not been able to secure a divorce from Klaus. The
this petition. latter could charge her for bigamy and could even involve Alfred,
who himself was still married. To avoid complications, Alfred
ISSUE: Whether or not Ong Ching Po could legally acquire the decided to live separately from Ederlina and cut off all contacts
property? with her.

HELD: No. Petition is dismissed. On October 15, 1985, Alfred wrote to Ederlinas father,
complaining that Ederlina had taken all his life savings and
RATIO: Ong Ching Po cannot legally claim ownership of the because of this, he was virtually penniless. He further accused
disputed property since the capacity to acquire private lands is the Catito family of acquiring for themselves the properties he
dependent on the capacity to acquire or hold lands of the public had purchased with his own money. He demanded the return of
domain. Private land may be transferred or conveyed only to all the amounts that Ederlina and her family had stolen and turn
individuals or entities “qualified to acquire lands of the public over all the properties acquired by him and Ederlina during their
domain”. Petitioner Ong Ching Po was a Chinese citizen and coverture.
therefore is disqualified from acquiring and owning real property.
Further, despite the documentary evidence provided by ISSUE: Whether the petitioner could recover the money used in
petitioners, they failed to provide evidence as to the genuineness purchasing the several properties
and due execution of the deed of sale. Likewise, the tax receipts,
tax declaration, rental receipts, deed of sale and TCT were in HELD: No, even if, as claimed by the petitioner, the sales in
Ong’s possession, these were not reflective of dominion or question were entered into by him as the real vendee, the said
ownership as even a mere administrator or manager may transactions are in violation of the Constitution; hence, are null
lawfully perform payment duties relative to his appointment as and void ab initio. A contract that violates the Constitution and
such. It was also noteworthy that the tax receipts and rental the law, is null and void and vests no rights and creates no
receipts were in the name of Parian’s husband. Hence, petition obligations. It produces no legal effect at all. The petitioner,
was dismissed. being a party to an illegal contract, cannot come into a court of
law and ask to have his illegal objective carried out. One who
50 Tacder NatRes

loses his money or property by knowingly engaging in a contract Lee vs Director of Lands
or transaction which involves his own moral turpitude may not FACTS:
maintain an action for his losses. To him who moves in  Sometime in March 1936, the Dinglasans sold to Lee Liong
deliberation and premeditation, the law is unyielding. The law (Chinese citizen) a parcel of land situated at the corner of
will not aid either party to an illegal contract or agreement; it Roxas Avenue and Pavia Street, Roxas City.
leaves the parties where it finds them  In 1993, Elizabeth Manuel-Lee and Pacita Yu Lee filed with
the RTC of Roxas City a petition for reconstitution of title
of the lot. (Alleging that the transfer certificate of title
Muller vs Muller issued to Lee Liong was lost or destroyed during World
FACTS: Elena Buenaventura Muller and Helmut Muller are War II.)
husband and wife in this case. They wed and resided in Germany  Petitioners Elizabeth and Pacita alleged that they were the
until they decided to permanently reside in the Philippines in widows of the deceased Lee Bing Hoo and Lee Bun
1992. By this time, they bought a house in Antipolo, Rizal using Ting, who were the heirs of Lee Liong, the owner of the
the proceeds that they got from selling the house the Helmut lot.
Muller inherited from his parents in Germany.  The RTC approved reconstitution of the lost or destroyed
The marriage, however, did not last. They were eventually certificate of title in the name of Lee Liong on the basis of
separated and Helmut Muller filed for separation of their an approved plan and technical description.
properties. The trial court then dissolved the absolute community  Solicitor General filed with the Court of Appeals a
of property and ordered the equal partition of their personal petition for annulment of the RTC decision alleging that
properties located within the country, excluding those acquired the RTC had no jurisdiction over the case.
by gratuitous title during marriage. As to the Antipolo property,  The Solicitor General contended that the petitioners were
the court held that it was acquired using the paraphernal funds of not the proper parties in the reconstitution of title, since
Helmut, however, he cannot recover said property, nor have a their predecessor-in-interest Lee Liong did not acquire title
right to recover the funds used to buy it since it was purchased in to the lot because he was a Chinese citizen and was
violation of Section 7, Article XII of the Constitution. Upon appeal, constitutionally not qualified to own the subject land.
the Court of Appeals granted Helmut Muller’s prayer for  CA declared the reconstitution void. Hence this petition.
reimbursement for the Antipolo property.  Elizabeth and Pacita emphasized that the ownership of
the land had been settled in two previous cases of the
ISSUE: WON Helmut Muller is entitled to reimbursement of the Supreme Court, where the Court ruled in favor of their
funds used to acquire the Antipolo property. predecessor-in-interest, Lee Liong.
 They also pointed out that they acquired ownership of the
HELD: NO. Respondent was aware of the constitutional land through actual possession of the lot and their
prohibition and expressly admitted his knowledge thereof to this consistent payment of taxes over the land for more than
Court. He declared that he had the Antipolo property titled in the sixty years.
name of petitioner because of the said prohibition. His attempt at  On the other hand, the Solicitor General submitted
subsequently asserting or claiming a right on the said property that the decision in the reconstitution case was void;
cannot be sustained. Thus, in the instant case, respondent otherwise, it would amount to circumventing the
cannot seek reimbursement on the ground of equity where it is constitutional proscription against aliens acquiring
clear that he willingly and knowingly bought the property despite ownership of private or public agricultural lands.
the constitutional prohibition. To allow reimbursement would in
effect permit respondent to enjoy the fruits of a property which ISSUES:
he is not allowed to own. Thus, it is likewise proscribed by law. 1. WON Lee Liong has the qualification to own land in the
Philippines. NO
51 Tacder NatRes

2. WON the reconstitution was valid. NO transferee is rendered valid.”

HELD: Thus, the subsequent transfer of the property to qualified


1. Lee Liong was not qualified but the ownership of the Filipinos may no longer be impugned on the basis of the invalidity
lot was already acquired by Filipino citizens Lee Liong was of the initial transfer. The objective of the constitutional provision
disqualified to acquire the land under the 1935 to keep our lands in Filipino hands has been achieved.
Constitution. The sale of the land in question was
consummated sometime in March 1936, during the Incidentally, it must be mentioned that reconstitution of the
effectivity of the 1935 Constitution. original certificate of title must be based on an owner’s duplicate,
secondary evidence thereof, or other valid sources of the title to
Under the 1935 Constitution aliens could not acquire private be reconstituted.
agricultural lands, save in cases of hereditary succession. Thus,
Lee Liong, a Chinese citizen, was disqualified to acquire the land 2. Reconstitution was void for lack of factual support
in question. In this case, reconstitution was based on the plan and technical
description approved by the Land Registration Authority. This
The fact that the Court did not annul the sale of the land to an renders the order of reconstitution void for lack of factual
alien did not validate the transaction. It was still contrary to the support. A judgment with absolutely nothing to support it is void.
constitutional proscription against aliens acquiring lands of the
public or private domain. As earlier mentioned, a reconstitution of title is the re-
issuance of a new certificate of title lost or destroyed in its
The proper party to assail the sale is the Solicitor General. original form and condition. It does not pass upon the ownership
of the land covered by the lost or destroyed title.
This was what was done in this case when the Solicitor
General initiated an action for annulment of judgment of Any change in the ownership of the property must be the subject
reconstitution of title. While it took the Republic more than of a separate suit. Thus, although petitioners are in possession of
sixty years to assert itself, it is not barred from initiating such the land, a separate proceeding is necessary to thresh out the
action. Prescription never lies against the State. issue of ownership of the land.

The land is now in the hands of Filipinos. 1.CONSTITUTIONAL LAW; PROSCRIPTION ON SALE OF REAL
ESTATE TO ALIENS; DOCTRINE OF PARI DELICTO APPLICABLE TO
The original vendee, Lee Liong, has since died and the land has VENDOR AND VENDEE THEREIN; CASE AT BAR. — "In sales of real
been inherited by his heirs and subsequently their heirs, estate to aliens incapable of holding title thereto by virtue of the
petitioners herein. Petitioners are Filipino citizens, a fact the provisions of the Constitution both the vendor and the vendee
Solicitor General does not dispute. are deemed to have committed the constitutional violation and
being thus in pari delicto the courts will not afford protection to
The constitutional proscription on alien ownership of lands of the either party." The proper party to assail the sale is the Solicitor
public or private domain was intended to protect lands from General. This was what was done in this case when the Solicitor
falling in the hands of non-Filipinos. In this case, however, there General initiated an action for annulment of judgment of
would be no more public policy violated since the land is in the reconstitution of title. While it took the Republic more than sixty
hands of Filipinos qualified to acquire and own such land. years to assert itself, it is not barred from initiating such action.
Prescription never lies against the State. Although ownership of
“If land is invalidly transferred to an alien who subsequently the land cannot revert to the original sellers, because of the
becomes a citizen or transfers it to a citizen, the flaw in the doctrine of pari delicto, the Solicitor General may initiate an
original transaction is considered cured and the title of the action for reversion or escheat of the land to the State, subject to
52 Tacder NatRes

other defenses, as hereafter set forth. In this case, subsequent nd


usufruct of the 2 part, 1/3 was given to the widow and 2/3 to
circumstances militate against escheat proceedings because the Wanda de Wrobleski, an Austrian. The grandnephews opposed on
land is now in the hands of Filipinos. The original vendee, Lee the ground that usufruct to Wanda is void because it violates the
Liong, has since died and the land has been inherited by his heirs constitutional prohibition against the acquisition of lands by
and subsequently their heirs, petitioners herein. Petitioners are aliens.
Filipino citizens, a fact the Solicitor General does not dispute.
ISSUE:
2.ID.; ID.; ID.; EFFECT OF SUBSEQUENT ACQUISITION OF
PHILIPPINE CITIZENSHIP BY TRANSFEREE; CASE AT BAR. — The WON the ground for the opposition is correct.
constitutional proscription on alien ownership of lands of the
public or private domain was intended to protect lands from HELD:
falling in the hands of non-Filipinos. In this case, however, there
would be no more public policy violated since the land is in the No, it is not correct.
hands of Filipinos qualified to acquire and own such land. "If land
is invalidly transferred to an alien who subsequently becomes a The SC held that the Constitutional provision which
citizen or transfers it to a citizen, the flaw in the original enables aliens to acquire private lands does not extend to
transaction is considered cured and the title of the transferee is testamentary succession for otherwise the prohibition will be
rendered valid." Thus, the subsequent transfer of the property to for naught and meaningless. The SC upheld the usufruct in favor
qualified Filipinos may no longer be impugned on the basis of the of Wanda because although it is a real right, it does not vest title
invalidity of the initial transfer. The objective of the constitutional to the land in the usufructuary and it is the vesting of title to land
provision to keep our lands in Filipino hands has been achieved. in favor of aliens which is proscribed by the Constitution.
ASCTac
Republic vs CA
3.CIVIL LAW; LAND REGISTRATION; RECONSTITUTION OF TITLE; On June 17, 1978, respondent spouses bought Lots, as their
VALID SOURCES OF THE TITLE TO BE RECONSTITUTED; CASE AT residence with a total area of 91.77 sq. m. situated in San Pablo
BAR. — Incidentally, it must be mentioned that reconstitution of City, from one Cristeta Dazo Belen. At the time of the purchase,
the original certificate of title must be based on an owner's respondent spouses where then natural-born Filipino citizens.
duplicate, secondary evidence thereof, or other valid sources of
the title to be reconstituted. In this case, reconstitution was On February 5, 1987, the spouses filed an application for
based on the plan and technical description approved by the registration of title of the two (2) parcels of land before the RTC of
Land Registration Authority. This renders the order of San Pablo City. This time, however, they were no longer Filipino
reconstitution void for lack of factual support. A judgment with citizens and have opted to embrace Canadian citizenship through
absolutely nothing to support it is void. naturalization.

RTC rendered a decision confirming the title of the Spouses.


Republic filed an opposition to the decision of RTC. On appeal, CA
Ramirez vs Vda de Ramirez affirmed the decision of RTC.
FACTS:
Republic submits that private respondents have not acquired
Jose Ramirez a Filipino, died in Spain leaving only his widow proprietary rights over the subject properties before they
Marcelle Ramirez, a French. In the project partition, the property acquired Canadian citizenship through naturalization to justify
was divided into 2 parts: 1st part to the widow, and 2nd part to the the registration thereof in their favor. It maintains that even
grandnephews the naked ownership. Furthermore, as to the privately owned unregistered lands are presumed to be public
53 Tacder NatRes

lands under the principle that lands of whatever classification


belong to the State under the Regalian doctrine. Thus, before the Sec. 8. Notwithstanding the provisions of Section 7 of this
issuance of the certificate of title, the occupant is not in the Article, a natural-born citizen of the Philippines who has
juridical sense the true owner of the land since it still pertains to lost his Philippine citizenship may be a transferee of
the State. Petitioner further argued that it is only when the court private lands, subject to limitations provided by law.
adjudicates the land to the applicant for confirmation of title (Emphasis supplied)
would the land become privately owned land, for in the same
proceeding, the court may declare it public land, depending on Section 8, Article XII of the 1987 Constitution above quoted is
the evidence. similar to Section 15, Article XIV of the then 1973 Constitution
which reads:
ISSUE: WON a foreign national apply for registration of title over
a parcel of land which he acquired by purchase while still a Sec. 15. Notwithstanding the provisions of Section 14 of this
citizen of the Philippines, from a vendor who has complied with Article, a natural-born citizen of the Philippines who has lost his
the requirements for registration under the Public Land Act (CA citizenship may be a transferee of private land, for use by him as
141)? YES his residence, as the Batasang Pambansa may provide.

HELD: Pursuant thereto, Batas Pambansa Blg. 185 was passed into
Since the spouses’ predecessor in interest has been in law, the relevant provision of which provides:
open, continuous and exclusive possession for at least 30 years
of alienable public land, such possession ipso jure converts the Sec. 2. Any natural-born citizen of the Philippines who has lost his
same to private property (Recall ruling of Director of Lands Philippine citizenship and who has the legal capacity to enter into
v. IAC). This means that occupation and cultivation for more than a contract under Philippine laws may be a transferee of a private
30 years by an applicant and his predecessors-in-interest, vest land up to a maximum area of one thousand square meters, in
title on such applicant so as to segregate the land from the mass the case of urban land, or one hectare in the case of rural land,
of public. to be used by him as his residence. In the case of married
couples, one of them may avail of the privilege herein granted;
What is important is that private respondents were formerly Provided, That if both shall avail of the same, the total area
natural-born citizens of the Philippines, and as transferees of a acquired shall not exceed the maximum herein fixed.
private land, they could apply for registration in accordance with In case the transferee already owns urban or rural lands for
the mandate of Section 8, Article XII of the Constitution. residential purposes, he shall still be entitled to be a transferee of
an additional urban or rural lands for residential purposes which,
In the case at bar, the spouses were undoubtedly natural- when added to those already owned by him, shall not exceed the
born Filipino citizens at the time of the acquisition of the maximum areas herein authorized.
properties and by virtue thereof, acquired vested rights thereon,
tacking in the process, the possession in the concept of owner From the adoption of the 1987 Constitution up to the present, no
and the prescribed period of time held by their predecessors-in- other law has been passed by the legislature on the same
interest under the Public Land Act. subject. Thus, what governs the disposition of private lands in
favor of a natural-born Filipino citizen who has lost his Philippine
But what should not be missed in the disposition of this citizenship remains to be BP 185.
case is the fact that the Constitution itself allows private
respondents to register the contested parcels of land in Even if private respondents were already Canadian citizens at the
their favor. Sections 7 and 8 of Article XII of the time they applied for registration of the properties in question,
Constitution contain the following pertinent provisions, to said properties as discussed above were already private lands;
wit: consequently, there could be no legal impediment for the
54 Tacder NatRes

registration thereof by respondents in view of what the NOW Director of lands instituted to
Constitution ordains. The parcels of land sought to be registered 1. declare null and void Free Patent issued in the name of
no longer form part of the public domain. They are already defendant Isagani Du Timbol;
private in character since private respondents' predecessors-in- 2. to order the aforesaid defendant to surrender the owner's
interest have been in open, continuous and exclusive possession duplicate of O.C.T. and the
and occupation thereof under claim of ownership prior to June 12, 3. defendant Register of Deeds to cancel the same;
1945 or since 1937. The law provides that a natural-born citizen 4. to decree the reversion of the land in question to the
of the Philippines who has lost his Philippine citizenship may be a mass of public domain
transferee of a private land up to a maximum area of 1,000 The land was originally applied for by Precila Soriain 1966, who
sq.m., if urban, or one (1) hectare in case of rural land, to be transferred her rights to the land and its improvements to
used by him as his residence (BP 185). defendant Isagani Du Timbol who filed his application on1969,
In 1969, free Patent No. was issued and City, (O.C.T.) No. P-2508
It is undisputed that private respondents, as vendees of a private in the name of defendant Isagani Du Timbol.
land, were natural-born citizens of the Philippines. For the In 1971, the Republic of the Philippines, at the instance of the
purpose of transfer and/or acquisition of a parcel of Bureau of Forestry, filed a complaint in to declare free patent and
residential land, it is not significant whether private Original Certificate of Title of Isagani Du Timbol null and void ab
respondents are no longer Filipino citizens at the time initio and to order the reversion of the land in question to the
they purchased or registered the parcels of land in mass of public domain.
question. What is important is that private respondents
were formerly natural-born citizens of the Philippines, Grounds
and as transferees of a private land, they could apply for  land covered thereby is a forest or timber land which is
registration in accordance with the mandate of Section 8, not disposable under the Public Land Act;
Article XII of the Constitution. Considering that private  in a reclassification of the public lands in the vicinity of
respondents were able to prove the requisite period and the land in question made by the Bureau of Forestry on
character of possession of their predecessors-in-interest over the March 7, 1958, was reverted to the category of public
subject lots, their application for registration of title must forest,
perforce be approved.  the application for free patent by Isagani Du Timbol more
than eleven years thereafter were obtained fraudulently
as private respondent Isagani Du Timbol never occupied
and cultivated the land applied for.
RTC -dismissed the complaint on the ground that Certificate of
Remedies to Recover Private Land from Disqualified Alien: Action for Title based on the patent had became indefeasible in view of the
reversion lapse of the one-year period prescribed under Section 38 of the
a) Who can initiate Land Registration Act for review of a decree of title on the ground
b) When proper. Improper of fraud.
c) Reversion vs Action for Nullity of Land Titles CA- grants the petition on the ground that the area covered by
the patent and title is not a disposable public land, it being a part
of the forest zone and, hence the patent and title thereto are null
Who can initiate and void.

Republic vs Animas Issue:WON LAND CAN BE REVERTED BACK TO THE STATE


ORIGINAL CASE: Republic of the Philippines, Plaintiff, vs. Isagani
Du Timbol and the Register of Deeds of General Santos City, Held: YES. A certificate of title fraudulently secured is null and
55 Tacder NatRes

void ab initio when fraud consisted in misrepresenting that the thereof, for to hold that a title may become indefeasible by
land covered by the application is part of the public domain when registration, even if such title had been secured through fraud or
it is not in violation of the law, would be the height of absurdity.
As a general rule, timber or forest lands are not alienable or Registration should not be a shield of fraud in securing title.
disposable under either the Constitution of 1935 or the
Constitution of 1973. It is the state is seeking the cancellation of the title,
therefore, it has not become indefeasible for prescription
When the defendant Isagani Du Timbol filed his application for cannot be invoked against the state. A title founded on
free patent over the land in question on June 3, 1969, the area in fraud may be cancelled, notwithstanding the lapse of one
question was not a disposable or alienable public land but a year from the issuance thereof, through a petition filed in
public forest. court by the Solicitor General.
It is the Bureau of Forestry that has jurisdiction and authority
over the demarcation, protection, management, reproduction, The land covered thereby may be reconveyed to the state
occupancy and use of all public forests and forest reservations in an action for reconveyance under Section 101 of
and over the granting of licenses for the taking of products Commonwealth Act 141 (Public Land Act).
therefrom, including stone and earth. Given that the area in
question is a forest or timber land is clearly established, then, the Isagani Du Timbol was never in possession of the property prior
Bureau of Forest Development has jurisdiction. to his filing the application, contrary to the provisions of law that
 the applicant must have been in possession or cultivation
Since it was Bureau of Lands who issued the patent and since it thereof for at least 30 years;
had no jurisdiction to issue a patent because the land involved  no existing signs of improvements found in the area in
was still inalienable forest land when granted, then it may be question as it is not under cultivation but covered with
plausibly contended that the patent title would be ab initio void, grasses, bushes and small trees;
subject to attack at any time by any party adversely affected.  that it is being used as ranch for grazing cows by the heirs
A patent is void at law if the of Hermogenes Chilsot;
 officer who issued the patent had no authority to do so  that no monuments were placed on the area surveyed
 If a person obtains a title by FAME under the Torrens which goes to show that there was no actual survey
System thereof;
 Director of Lands did not have jurisdiction over the same  inside the forest zone;
because it is a public forest, the grantee does not, by  the signature of then Acting District Land Officer Elias de
virtue of said certificate of title alone, become the owner Castro of South Cotabato has been forged
of the land illegally included. The above alleged circumstances are indicative of fraud and the
In the case at bar the party seeking the nullity of the title and misrepresentations of the applicant that he had been occupying
reversion of the land is the state itself which is specifically and cultivating the land and residing thereon are sufficient
authorized under Section 101 of the Public Land Act to grounds to ipso facto nullify the grant of the patent and title
initiate such proceedings as an attribute of sovereignty. under Section 91 of the Public Land Law which provides as
follows:
The defense of indefeasibility of a certificate of title issued A certificate of title that is void may be ordered cancelled.
pursuant to a free patent does not lie against the state in an A title will be considered void if it is procured through fraud, as
action for reversion of the land covered thereby when such land when a person applies for registration of the land under his name
is a part of a public forest or of a forest reservation. although the property belongs to another.
The lapse of the one year period within which a decree of title
may be reopened for fraud would not prevent the cancellation
56 Tacder NatRes

When proper/ improper


ISSUE

Republic vs Umali Whether or not the land under the new owners are obtained thru
FACTS forgery and fraud and subject to return the property to the State

The land situated in Tanza, Cavite which consists of HELD


78,865 square meters was originally purchased on installment
from the government on July 1, 1910 by Florentina Bobadilla, who We agree with the contention that there is no allegation in
allegedly transferred her rights thereto in favor of Martina, the complaint 13 filed by the petitioner that any one of the
Tomasa, Gregorio and Julio, all surnamed Cenizal, in 1922. 2 defendants was privy to the forged joint affidavit or that they had
Tomasa and Julio assigned their shares to Martina, Maria and acquired the subject land in bad faith. Their status as innocent
Gregorio. In 1971 these three assignees purportedly signed a transferees for value was never questioned in that pleading. Not
joint affidavit which was filed with the Bureau of Lands to support having been disproved, that status now accords to them the
their claim that they were entitled to the issuance of a certificate protection of the Torrens System and renders the titles obtained
of title over the said land on which they said they had already by them thereunder indefeasible and conclusive. The rule will not
made full payment. On the basis of this affidavit, the Secretary of change despite the flaw in TCT No. 55044.
Agriculture and Natural Resources executed Deed No. V-10910 Section 39 of the Land Registration Act clearly provided:
(Sale Certificate No. 1280) on September 10, 1971, in favor of Every person receiving a certificate of title in pursuance of a
the said affiants. decree of registration, and every subsequent purchaser of
registered land who takes a certificate of title for value in good
A complaint for reversion was filed on October 10, 1985 faith shall hold the same free of all encumbrance except those
when the registered owners of the land, following several noted on said certificate.
transfers, were Remedios Micla, Juan C. Pulido, and Rosalina, Luz The real purpose of the Torrens System of land
and Enrique Naval. They asked to return the property to the State registration is to quiet title to land; to put a stop forever to any
on the aforestated grounds of forgery and fraud. The plaintiff question of the legality of the title, except claims which were
claimed that Gregorio Cenizal having died on February 25, 1943, noted at the time of registration in the certificate, or which may
and Maria Cenizal on January 8, 1959, they could not have signed arise subsequent thereto. That being the purpose of the law, it
the joint affidavit dated August 9, 1971, on which Deed No. V- would seem that once the title was registered, the owner might
10910 (Sale Certificate No. 1280) was based. rest secure, without the necessity of waiting in the portals of the
court, or sitting in the "mirador de su casa," to avoid the
In their answer, Pulido and the Navals denied any possibility of losing his land.
participation in the join affidavit and said they had all acquired The difference between them and the private respondents
the property in good faith and for value. By way of affirmative is that the latter acquired the land in question not by direct grant
defenses, they invoked estoppel, laches, prescription and res but in fact after several transfers following the original sale
judicata. For her part, Miclat moved to dismiss the complaint, thereof to Bobadilla in 1910. The presumption is that they are
contending that the government had no cause of action against innocent transferees for value in the absence of evidence to the
her because there was no allegation that she had violated the contrary. The petitioner contends that it was Pedro Miclat who
plaintiff’s right, that the government was not the real party-in- caused the falsification of the joint affidavit, but that is a bare
interest because the subject land was already covered by the and hardly persuasive allegation, and indeed, even if true, would
Torrens system, and that in any event the action was barred by still not prove any collusion between him and the private
prescription or laches. respondents. The mere fact that Remedios Miclat was the
daughter and heiress of Miclat, without more, would not
57 Tacder NatRes

necessarily visit upon her the alleged sins of her father. difference between them and the private respondents is that the
The land being now registered under the Torrens system latter acquired the land in question not by direct grant but in fact
in the names of the private respondents, the government has no after several transfers following the original sale thereof to
more control or jurisdiction over it. It is no longer part of the Bobadilla in 1910. The presumption is that they are innocent
public domain or, as the Solicitor General contends — as if it transferees for value in the absence of evidence to the contrary.
made any difference — of the Friar Lands. The subject property
ceased to be public land when OCT No. 180 was issued to 3. ID.; ID.; SUBJECT LAND NO LONGER PART OF PUBLIC DOMAIN;
Florentina Bobadilla in 1910 or at the latest from the date it was LAND REGISTRATION ACT, NOW CALLED "PRIVATE REGISTRATION
sold to the Cenizals in 1971 upon full payment of the purchase DECREE" GOVERNS. — The land being now registered under the
price. As private registered land, it is governed by the provisions Torrens system in the names of the private respondents, the
of the Land Registration Act, now denominated the Property government has no more control or jurisdiction over it. It is no
Registration Decree, which applies even to the government. longer part of the public domain or, as the Solicitor General
contends — as if it made any difference — of the Friar Lands. The
1. LAND REGISTRATION; TORRENS SYSTEM; PROTECTS INNOCENT subject property ceased to be public land when OCT No. 180 was
TRANSFEREES FOR VALUE; TITLES OBTAINED RENDERED issued to Florentina Bobadilla in 1910 or at the latest from the
INDEFEASIBLE AND CONCLUSIVE. — There is no allegation in the date it was sold to the Cenizals in 1971 upon full payment of the
complaint filed by the petitioner that any one of the defendants purchase price. As private registered land, it is governed by the
was privy to the forged joint affidavit or that they had acquired provisions of the Land Registration Act, now denominated the
the subject land in bad faith. Their status as innocent transferees Property Registration Decree, which applies even to the
for value was never questioned in that pleading. Not having been government. The pertinent provision of the Land Registration Act
disproved, that status now accords to them the protection of the was Section 122, which read as follows: Sec. 122. "Whenever
Torrens System and renders the titles obtained by them public lands in the Philippine Islands belonging to the
thereunder indefeasible and conclusive. The rule will not change Government of the United States or to the Government of the
despite the flaw in TCT No. 55044. Section 39 of the Land Philippine Islands are alienated, granted, or conveyed to persons
Registration Act clearly provided: "Sec. 39. Every person or to public or private corporations, the same shall be brought
receiving a certificate of title in pursuance of a decree of forthwith under the operation of this Act and shall become
registration, and every subsequent purchaser of registered land registered lands." This should be related to Section 12 of the Friar
who takes a certificate of title for value in good faith shall hold Lands Act, providing thus: "Sec. 12. . . . upon the payment of the
the same free of all encumbrance except those noted on said final installment together with all accrued interest, the
certificate." Government will convey to such settler and occupant the said
land so held by him by proper instrument of conveyance, which
2. ID.; ID.; ID.; PIÑERO v. DIRECTOR OF LANDS, 57 SCRA 386, shall be issued and become effective in the manner provided in
NOT APPLICABLE IN CASE AT BAR. — The decision in Piñero v. section one hundred and twenty-two (Sec. 122) of the Land
Director of Lands is not applicable to the present proceeding Registration Act."
because the lands involved in that case had not yet passed to
the hands of an innocent purchaser for value. They were still held 4. ID.; ID.; ORIGINAL TRANSFER OF LAND, MERELY AVOIDABLE,
by the Piñeros. The action for reversion was filed by the NOT VOID AB-INITIO; LAND CEASES TO BE PUBLIC UPON
government against them as the original transferees of the REGISTRATION AND ISSUANCE OF CERTIFICATE OF TITLE. — The
properties in question. They were the direct grantees of the free petitioner errs in arguing that the original transfer was null and
patents issued by the government pursuant to which the void ab initio, for the fact is that it is not so. It was only voidable.
corresponding certificates of title were issued under the Torrens The land remained private as long as the title thereto had not
system. The fraud alleged by the government as a ground for the been voided, but it is too late to do that now. As the Court has
reversion sought was imputable directly to the Piñeros, who could held in Ramirez vs. Court of Appeals. (30 SCRA 301): "A
not plead the status of innocent purchasers for value.The certificate of title fraudulently secured is not null and void ab
58 Tacder NatRes

initio, unless the fraud consisted in misrepresenting that the land


is part of the public domain, although it is not. In such case the ISSUE:
nullity arises, not from the fraud or deceit, but from the fact that Whether or not the actual occupancy of a part of the land
the land is not under the jurisdiction of the Bureau of Lands. described in the instrument giving color of title sufficient to give
Inasmuch as the land involved in the present case does not title to the entire tract of land?
belong to such category. OCT No. 282-A would be merely
voidable or reviewable. . . . Once a patent is registered and the HELD:
corresponding certificate of title is issued, the land ceases to be The general rule is that possession and cultivation of a portion of
part of public domain and becomes private property over which a tract of land under the claim of ownership of all is a
the director of Lands has neither control nor jurisdiction. A public constructive possession of all, if the remainder is not in the
land patent, when registered in the corresponding Register of adverse possession of another.
Deeds, is a veritable Torrens Title, and becomes as indefeasible
as Torrens Title upon the expiration of one (1) year from the date The claimant has color of title; he acted in good faith and he has
of issuance thereof." open, peaceable, and notorious possession of a portion of the
property, sufficient to apprise the community and the world that
the land was for his enjoyment.

Possession in the eyes of the law does not mean that a man has
Pinero vs Director of Lands to have his feet on every square meter of ground before it can be
The general rule is that possession and cultivation of a portion of said that he is in possession. Ramos and his predecessor in
a tract of land under the claim of ownership of all is a interest fulfilled the requirements of the law on supposition that
constructive possession of all, if the remainder is not in the the premises consisted of agricultural public land.
adverse possession of another.
On the issue of forest land, Forest reserves of public land can be
FACTS: established as provided by law. When the claim of the citizen and
Restituo Romero gained possession of a considerable tract of the claim of the government as to a particular piece of property
land located in Nueva Ecija. He took advantage of the Royal collide, if the Government desires to demonstrate that the land is
Decree to obtain a possessory information title to the land and in reality a forest, the Director of Forestry should submit to the
was registered as such. court convincing proof that the land is not more valuable for
agricultural than for forest purposes.
Parcel No. 1 included within the limits of the possessory
information title of Romero was sold to Cornelio Ramos, herein In this case, the mere formal opposition on the part of the
petitioner. Attorney-General for the Director of Forestry, unsupported by
satisfactory evidence will not stop the courts from giving title to
Ramos instituted appropriate proceedings to have his title the claimant.
registered. Petitioner and appellant has proved a title to the entire tract of
Director of Lands opposed on the ground that Ramos had not land for which he asked for registration.
acquired a good title from the Spanish government. Registration in the name of the petitioner is hereby granted.

Director of Forestry also opposed on the ground that the first 1. Relevant Issue
parcel of land is forest land.
It has been seen however that the predecessor in interest to the - Whether or not the Director of Lands had authority to
petitioner at least held this tract of land under color of title. order the investigation of the patents and title
59 Tacder NatRes

fraud or mistake; as the case may be, in successfully obtaining


2. Relevant Decision these documents of title over the parcel of land claimed by
plaintiff. Private respondents allege in their complaint all the
- YES. The Director of Lands had authority to order the facts necessary to seek the nullification of the free patents as
investigation. well as the certificates of title covering Lot 1015 and Lot1017.
Clearly, they are the real parties in interest in light of their
3. Doctrine allegations that they have always been the owners and
possessors of the two (2) parcels of land even prior to the
- It is not only the right but the DUTY of the Director of issuance of the documents of title in petitioners’ favor, hence the
Lands to conduct investigation of any alleged fraud in latter could only have committed fraud in securing them.
securing a free patent, and the corresponding title to a
pubic land In the more recent case of Heirs of Ambrocio Kionisala v. Heirs of
- He may also file the corresponding court action for the Honorio Dacut, 30 the difference between an action for
reversion of the public land to the State declaration of nullity of land titles from an action for reversion
- Sec. 91 of the Public Land Act states: was more thoroughly discussed as follows:
“It shall be the duty of the Director of Lands, from time to time
and whenever he may deem it advisable, to make the necessary An ordinary civil action for declaration of nullity of free patents
investigations for the purpose of ascertaining whether the and certificates of title is not the same as an action for reversion.
material facts set out in the application are true, or whether they The difference between them lies in the allegations as to the
continue to exist and are maintained and preserved in good character of ownership of the realty whose title is sought to be
faith..” nullified. In an action for reversion, the pertinent allegations in
the complaint would admit State ownership of the disputed land.
Hence, in Gabila vs. Barriga [41 SCRA 131], where the plaintiff in
his complaint admits that he has no right to demand the
Reversion vs Action for Nullity of Land Titles cancellation or amendment of the defendant's title because even
if the title were canceled or amended the ownership of the land
embraced therein or of the portion affected by the amendment
Kionisala vs Dacut would revert to the public domain, we ruled that the action was
FACTS: for reversion and that the only person or entity entitled to relief
Private respondents filed a complaint for declaration of would be the Director of Lands. On the other hand, a cause of
nullity of titles, reconveyance and damages against petitioners action for declaration of nullity of free patent and certificate of
involving two parcels of land – Lot No. 1017 and Lot No. 1015. title would require allegations of the plaintiff's ownership of the
Said lots were registered in their respective names. They claimed contested lot prior to the issuance of such free patent and
absolute ownership of Lot 1015 and 1017 even prior to the certificate of title as well as the defendant's fraud or mistake, as
issuance of the corresponding free patents and certificates of the case may be, in successfully obtaining these documents of
title. title over the parcel of land claimed by plaintiff. In such a case,
ISSUE: Whether or not the complaint filed by private the nullity arises strictly not from the fraud or deceit but from the
respondents was sufficient. fact that the land is beyond the jurisdiction of the Bureau of
HELD: Lands to bestow and whatever patent or certificate of title
YES. A cause of action for declaration of nullity of free obtained therefore is consequently void ab initio. The real party-
patent and certificate of title would require allegations of the in-interest is not the State but the plaintiff who alleges a pre-
plaintiff’s ownership of the contested lot prior to the issuance of existing right of ownership over the parcel of land in question
such free patent and certificate of title as well as the defendant’s even before the grant of title to the defendant. . . .
60 Tacder NatRes

Evangelista vs Santiago

Roco vs Gineda

Republic vs Hachero

Republic vs Espinosa

You might also like